This is “Modern Theories of Organizational Communication”, chapter 4 from the book An Introduction to Organizational Communication (v. 0.0). For details on it (including licensing), click here.

For more information on the source of this book, or why it is available for free, please see the project's home page. You can browse or download additional books there. To download a .zip file containing this book to use offline, simply click here.

Has this book helped you? Consider passing it on:
Creative Commons supports free culture from music to education. Their licenses helped make this book available to you.
DonorsChoose.org helps people like you help teachers fund their classroom projects, from art supplies to books to calculators.

Chapter 4 Modern Theories of Organizational Communication

Expanding Your View

Up to now, your introduction to organizational communication has been fairly straightforward. The definition of an “organization” presented in Chapter 1 "Introduction to Organizational Communication" emphasized aspects of the workplace that you probably expected—structure, goals, personnel, etc., and the definition of “communication” featured elements that can be easily understood—source, message, channel, receiver. Then in Chapter 3 "Classical Theories of Organizational Communication" we explored classical theories of organizational communication that are driven by attitudes you have likely encountered on the job—your supervisor’s desire for machine-like efficiency, your company’s view of employees as “human resources” that must be beneficially managed.

In this chapter, however, we are going to complicate these pictures. Yet by expanding your view of “organization” and “communication,” you can better understand the often bewildering and messy realities of everyday life on the job. Modern theories of organizational communication—the subject of this chapter—are driven by a recognition that “real life” in the workplace seldom conforms to such ideals as smoothly operating hierarchies and clearly transmitted messages.

For example, has your boss ever yelled at you? Irrational behavior can be difficult to square with classical theories of organization and communication. Though a message is obviously being transmitted from a source (your boss) to a receiver (you), insults generate far more mental stimulation than is necessary and, in fact, introduce inaccuracies that will likely cause you to misinterpret the message. Cursing hardly reflects the scientific management advocated by Frederick Taylor, the impersonal environment espoused by Max Weber, and the precise wording of commands favored by Henri Fayol. So by these lights, your boss’s yelling is inefficient and counterproductive. Neither are curses and insults conducive to good human relations in the workplace—or to satisfying your hierarchy of needs, or giving you positive motivation and enjoyment in your job, or encouraging your involvement in workplace decisions. By all these accounts, yelling and cursing is bad management—and yet, as we will see in Chapter 13 "Technology in Organizations", it occurs daily in organizations worldwide. One study estimated that 37 percent of workers will be subjected to workplace bullying in the course of their careers. In the United States alone, that amounts to more than 56 million people.Namie, G. (2010). Workplace Bullying Institute 2010 U.S. workplace bullying survey. Retrieved April 22, 2012, from http://www.workplacebullying.org/wbiresearch/2010-wbi-national-survey

In this chapter, we will expand our view of organization and communication in ways that allow us to consider some new perspectives: Did your boss yell to assert power over you? Was this assertion of power rooted in historical prejudices or in attitudes that prevail in the surrounding society? Is aggression tied to the very nature of organizing itself? Or is aggression rooted in the culture of your particular organization, a pattern that employees past and present have established, so that yelling is way that people “make sense” of a super-competitive work environment?

Learning about modern theories of organizational communication will help us explore such questions. Before describing these theories, however, we must first revisit the assumptions that we have built up in the preceding chapters. This is because modern theories are often based on different assumptions about the nature of organizations and communication than are classical theories. We are not asking you to discard classical thinking; the theories developed by Taylor, Weber, Fayol, and scholars in the human relations and human resources traditions address real issues in the workplace and remain influential. Rather, we are asking you to build on the foundation of classical theory and now expand your view.

4.1 Rethinking the Organization

Learning Objectives

  1. Differentiate among the four approaches to theorizing about organizations: postpositive, interpretive, critical, and postmodern.
  2. Understand how these approaches are driven by three decisions: about ontology (how things exist), epistemology (how things are known), and axiology (what is worth knowing).

In Chapter 1 "Introduction to Organizational Communication" we read fifteen representative definitions of “organization” (see Table 1.1). All fifteen contained one or more of the following words (or their variants): system, structure, unit, collective, pattern, coordination.

When we think of a “system” or “structure” we usually think of an object, a thing that exists independently of the people in it. People come and go, but the system endures. Yet when we think of a system as a “thing” we are thinking metaphorically. As noted in the introduction to Chapter 3 "Classical Theories of Organizational Communication", a metaphor is not a literal description but rather a linguistic means to grasp a concept by comparing it to something from the real world. Thus, we think of time as an object in such metaphors as “time flies” and “time is money.”Lakoff, G., & Johnson, M. (1980). Metaphors we live by. Chicago: University of Chicago Press. In the same way, although a system is not an actual, literal, physical object which you can hold in your hands, thinking of it that way helps you picture how a system functions.

Similarly, when you think of an “organization” you probably think of it as an object with its own existence. Most people do. A corporation, for example, is considered a “person” under United States law for purposes ranging from taxation to free speech. Clearly, however, thinking of an organization as an object is a metaphor. Nevertheless, the way that we conceptualize an “organization” has very real consequences for organizational communication theory.

Three Decisions about Theory

The assumption that an organization is an object with an independent existence—that is to say, it has an “objective” rather than “subjective” reality—is characteristic of the postpositiveAn approach to organizational communication which holds that organizations have objective existences. Since the imperative to optimize performance governs the organization, individual mindsets ultimately are superfluous. Organizational behaviors are therefore best studied in the aggregate through empirical observations that leads to measurable results. (sometimes called positivist or functionalist) tradition in organizational communication scholarship. Below we will review the postpositive perspective and then, as alternatives, introduce the interpretiveAn approach to organizational communication which holds that organizations have subjective existences and, in fact, are constitued through their members’ communication. As such, it is not enough to observe aggregate behaviors; individual mindsets must be also be interpreted., criticalAn approach to organizational communication that employs theory as a framework to expose the hidden power structures in organizations and the ways that dominant interests distort meaning, thought, consciousness, and communicative action to maintain their domination by marginalizing alternative expressions., and postmodernAn approach to organizational communication which holds that organizations come into existence as temporary combinations of interests against the threatening fluidity of larger historical and cultural discourses. As a reflection of these discourses, the organization is a “text” that can be “read” in order to trace back how its hidden power relations were formed. perspectives on organizations. Each approach to how we conceive of organizations involves different assumptions. For theorists, their assumptions imply three decisions: ontology, epistemology, and axiology.

Ontology

Our ontologyPhilosophy of how things have being. Some theorists believe an organization exists independently from how people perceive it; others believe an organization exists only in relation to the perceptions of its people or in relation to society. is how we think about the nature of being. Do we think of an organization as having its own existence and own behaviors that continue independently of the various managers and employees who come and go over time? Or do we believe these individuals create and continuously re-create the organization and therefore drive its behaviors? Or is our concept of the organization, and our expectations for the form it should take and what it should do, determined by larger historical and cultural forces?

Epistemology

Our epistemologyPhilosophy of how things are known. Some researchers believe it is sufficient to observe and measure an organization’s aggregate behaviors; others believe that the mindsets and interactions of individuals must also be interpreted. is our philosophy of how things come to be known. Do we believe that knowledge about an organization is attained by observing collective actions and measuring aggregate behaviors? Or by listening to individual members of an organization and interpreting organizational life on their terms? Or by tracing the historical and cultural forces that have shaped people’s expectations for what an organization should be and the roles that managers and employees should play?

Axiology

Our axiologyPhilosophy of what is worth knowing. Some researchers only accept knowledge gained empirically through observation and measurement of aggregate behaviors; others believe that people’s perceptions must be analyzed. is what we believe is worth knowing, a decision that involves a value judgment. Many social scientists believe that only empirical evidence, or what can be directly and impartially observed and measured, is worth knowing. Others ask whether any research is truly value-neutral or can be based on “just the facts.” Does not the choice of research method influence what is found? Indeed, is not a decision to accept only what can be measured in itself a value judgment? Where some scholars strive to produce impartial knowledge, which organizational management can use to improve results, others believe such a goal implicitly supports the current system and those in power. Furthermore, where some researchers measure aggregate responses, others strive to hear organizational members on their own terms while giving voice to the powerless and thereby effecting social change.

All three issues—ontology, epistemology, and axiology—are deeply implicated in both classical and modern theories of organizational communication.

Four Questions about Organizations

What is now called the postpositive (or sometimes positivist or functionalist) approach dominated organization studies through the 1970s.Redding, C., & Tompkins, P. (1988). Organizational communication—Past and future tenses. In G. Goldhaber & G. Barnett (Eds.), Handbook of organizational communication (pp. 5-34). Norwood, NJ: Ablex. Most scholars in the field took it for granted that organizations could, and should, be studied through scientific methods. Then in 1979, Gibson Burrell and Gareth Morgan published an influential work that proposed new paradigms for organizational studies.Burrell, G., & Morgan, G. (1979). Sociological paradigms and organizational analysis. London: Heinemann. They started with four basic questions about the assumptions of social science:

  • Do social realities, such as organizations, have objective or subjective existence; i.e., do they exist on their own or only in people’s minds?
  • Can one understand these social realities through observation or must they be directly experienced?
  • Is knowledge best gained by scientific methods or by participating in a social reality from the inside?
  • Do people have free will or are they determined by their environments?

According to Burrell and Morgan, these issues boil down into two fundamental debates: whether social realities exist objectively or subjectively, and whether their basic state is order or conflict (what Burrell and Morgan called “regulation” or “radical change”). These two questions form the axes of a 2 × 2 matrix which we have adapted from Burrell and Morgan and show in Figure 4.1 "Approaches to Organizations: Burrell & Morgan" below.

Figure 4.1 Approaches to Organizations: Burrell & Morgan

During the 1980s and beyond, scholars used Burrell and Morgan’s matrix to flesh out new approaches for organizational research.For example, see Redding & Tompkins, op. cit.; Putnam, L. (1982). Paradigms for organizational communication research. Western Journal of Speech Communication, 46, 192-206. More recently, Stanley Deetz took stock of how the field has developed since Burrell and Morgan’s original analysis.Deetz, S. (2001). Conceptual foundations. In F. M. Jablinb & L. L. Putnam (Eds.), The new handbook of organizational communication: Advances in theory, research, and methods (pp. 3-46). Thousand Oaks, CA: Sage. See also Deetz, S. (1994). Representative practices and the political analysis of corporations. In B. Kovacic (Ed.), Organizational communication: New perspectives (pp. 209-242). Albany: State University of New York Press. He proposed a new matrix that retains the order-versus-conflict axis (what Deetz called “consensus” versus “dissensus”) but substituted a new second axis. For Deetz, the two basic questions are: (1) is order or conflict the natural state of an organization; and (2) should researchers apply “knowledge to,” or derive “knowledge from,” an organization—that is, should they start with an existing theory and see how an organization might fit, or study an organization on its own terms? (Deetz called these the “elite/a priori” versus the “local/emergent” approach.) By adapting Deetz’s two questions, we can construct the matrix shown in Figure 4.2 "Approaches to Organizations: Deetz" below. Though Deetz preferred the terms “normative” and “dialogic” for postpositive and postmodern, we use the latter terms because they are widely recognized among organizational communication scholars.

Figure 4.2 Approaches to Organizations: Deetz

Thus, postpositive researchers believe that order is the natural state of an organization, and postpositive researchers look to fit a given organization into an existing theory of how order is produced. Interpretive researchers likewise believe that order is the natural state of an organization, but they study each organization on its own terms and how its members establish patterns of conduct. Critical researchers, on the other hand, believe that conflict is the natural state of an organization and bring existing theories about conflicts over power to their analyses of a given organization. Postmodern researchers also believe that conflict is the natural state of an organization, but they look to deconstruct the particular power relations that have emerged in a given organization.

In our view, two questions originally posed by Burrell and Morgan can be recast to provide one more helpful framework for understanding the differences between the postpositive, interpretive, critical, and postmodern approaches. Those questions are: (1) what is the nature of reality; and (2) what is the source of structure? As to the first question, Steven Corman contrasted the realistThe belief that a thing, including a social phenonemon such as an organization, has an existence independent from people’s perception of it. belief “that things (including social phenomena) have a reality that is independent of their being perceived by someone,” with the relativistThe belief that a thing, including a social phenomenon such as an organization, has an existence only in relation to some point of view. view that “things (especially social phenomena) exist only in relation to some point of view.”Corman, S. R. (2005). Postpositivism. In S. May & D. K. Mumby (Eds.), Engaging organizational communication theory and research: Multiple perspectives (pp. 15-34). Thousand Oaks, CA: Sage; pg. 25. As to the second question, theorists draw a distinction between structure and agencyThe debate among theorists about whether people are determined by their environments (structure) or have free will (agency).. As Burrell and Morgan noted, some theorists believe that people are determined by their environments (structures), while others hold that people have free will (agency). Applied to an organization, the question becomes whether its structures are determined by socio-historical processes that operate outside the organization or are created through agency of its members. Again, these two questions about reality and structure can form the axes of the matrix shown in Figure 4.3 "Approaches to Organizations".

Figure 4.3 Approaches to Organizations

Thus, postpositive theorists believe the structures established by an organization’s members literally take on a life of their own, attaining an objective reality that endures independently over time. Critical theorists also believe that organizational structures have a fixed reality, but they see these structures originating in socio-historical processes that operate outside the organization. On the other hand, interpretive theorists believe that an organization has a subjective reality and exists only in relation to the viewpoints of the people inside the organization. Postmodern theorists also believe an organization has a subjective reality, but they see this reality existing in relation to socio-historical points of view that originate outside the organization.

As we will describe at the conclusion of this section, your task is not to choose one “best” approach to organizational communication over another, but to appreciate and draw from each. Toward that end, let us now explore the respective approaches in more detail. In so doing, we will concentrate on their respective ontologies, epistemologies, and axiologies. For the moment, we are only describing the approaches, and not specific theories within each approach.

Postpositive Approach

In the classic disaster-move spoof Airplane!, passengers and crew start to become mysteriously ill. A doctor on board exclaims, “This woman has got to be taken to a hospital.” The chief flight attendant anxiously asks, “A hospital! What is it?” To this the doctor replies, “It’s a big building with patients. But that’s not important right now.”Davison, J., Koch, H. W. (Producers), & Abrahams, J. Zucker, D. & Zucker, J. (Directors). (1980). Airplane! [Motion picture]. United States: Paramount. Similarly, we will not spend much time here discussing the difference between positivism and postpositivism. That’s not important right now. Suffice it to say that, as Steven Corman explained, where positivistic scientists of the early twentieth century took the antirealist position that existence only mattered insofar as what meets the eye, today’s postpositivists hold the realist belief that reality exists independently of perception.Corman, op cit.

Lex Donaldson succinctly captured this perspective by suggesting that “in any situation, to attain the best outcome, the decision-makers must choose the structure that best fits that situation . . . with the ideas of the decision-makers making no independent contribution to the explanation of the structure.” In other words, since an organization can survive only if it performs well, managers are ultimately forced to choose the course of action that gets the best results. Even when managers choose lesser options, the resulting performance deficit creates its own pressure to either correct the mistake or go out of business. In the end, “the consciousness of the actors [is] superfluous” because “there will be an irresistible tendency for organization managers to choose options that conform to the situational imperative . . . with no moderation by managerial ideas.”Donaldson, L. (2003). Organization theory as a positive science. In H. Tsoukas & C. Knudsen, The Oxford handbook of organizational theory: Meta-theoretical perspectives (pp. 39-62). Oxford, UK: Oxford University Press; pgs. 44-45. The same holds true when managers communicate; they are forced, in the end, to choose messages and channels that best contribute to the bottom line. In the postpositive view, then, the purpose of organizational communication is instrumental—that is, an instrument for achieving results. Accurate messages and precise instructions are therefore seen as the best guarantors of optimal performance.

Given this conception of the organization, we can see how postpositivism fits together with its own distinctive ontology (its belief in how things exist), epistemology (its belief in how things are known), and axiology (its belief in what is worth knowing). Because the organization has an independent reality, its imperatives—to survive, to get the best results—drive what people do (rather than vice versa). And because individual mindsets ultimately do not matter, then researchers learn about an organization by observing its aggregate behaviors.

Thus, for example, Frederick Taylor’s classical theory of scientific management is based on the assumption that what comes out of the organization is a function of went in. This idea is expressed in popular acronym GIGO for “garbage in, garbage out.” The task for managers who observe poor output is to scientifically adjust input. If so, their metaphorical well-oiled machine can work at maximum capacity. The goal, not only for managers but for postpositive organizational theorists, is to move from description and explanation to prediction of causes and effects—which implies the ability to control effects by adjusting causes. Figure 4.4 "Goals for Postpositive Research" below illustrates this progression. In organizational research, studies undertaken from a postpositive perspective are often intended to generate knowledge that can be applied to improving management practices.

Figure 4.4 Goals for Postpositive Research

Even as Frederick Taylor did a century ago with his time and motion studies, those today who study organizations from a postpositive perspective see themselves as social scientists. They practice nomotheticAn approach to knowledge that emphasizes scientific testing of hypotheses and employs quantitative tests, such as surveys, which generate numerical data. The ultimate goal of nomothetic research is to discover laws that can be generally applied across many cases. research methods that emphasize scientific testing of hypotheses and employ quantitative methods, such as surveys and experiments, which generate numerical data. For postpositive researchers, this is the only data worth knowing; they disfavor the ideographicAn approach to knowledge that takes each case on its own terms by considering qualitative data such as ethnographic fieldwork, interviews, journals, and diaries. data generated by such qualitative methods as ethnographic fieldwork, interviews, journals, and diaries because postpositivists find these methods inherently subjective and unable to describe what they perceive as the objective reality of organizational communication. The ultimate goal of nomothetic research is to discover general laws that are applicable across different cases. Classical examples of the nomothetic approach to research are described in Chapter 3 "Classical Theories of Organizational Communication", including the Hawthorne Studies of Elton Mayo and the pajama factory study of Kurt Lewin.

As an interesting caveat to this discussion, people who conduct what we are labeling “postpositive” research generally do not describe their work as such. Because the field of organizational communication research grew out of the social-psychological and business research of the first part of the 20th century, today’s postpostive researchers follow their counterparts in fields like industrial psychology or organizational behavior and categorize themselves as social scientists. While social-scientific researchers in organizational communication do not discount what other researchers in the larger field of organizational communication are doing, they do see themselves and their research as very distinct from the work of interpretive, critical, and postmodern researchers. As Patric Spence and Colin Baker noted in their article examining the types of organizational communication research published within the field, postpositive research still accounts for almost half of the research published today.Spence, P. R., & Baker, C. R. (2007). State of the method: An examination of level of analysis, methodology, representation and setting in current organizational communication research. Journal of the Northwest Communication Association, 36, 111–124.

Table 4.1 "The Postpositive Approach" summarizes our discussion about the components of the postpositive perspective on organizational communication.

Table 4.1 The Postpositive Approach

Ontology (how things exist) Epistemology (how things are known) Axiology (values for research) Purpose of Org. Communication
Realism Observation Intervention Instrumental
Organizations have an objective existence that is independent of the people in them. People come and go but the organization endures. Since people ultimately must choose actions that get the best organizational results, individual mindsets do not matter. Thus, to learn about an organization it is sufficient to observe its aggregate behaviors. Social-scientific research generates knowledge that can be used to make predictive theories and applied to management practices. Organizational imperatives that force people to choose the most effective actions apply to communication actions. Thus, accurate and precise communications are most effective.

Interpretive Approach

Where postpositive theorists believe the organization drives what its people do, interpretive theorists believe the reverse: that people drive what their organizations do—and, in fact, what their organizations are. As Dennis Mumby and Robin Clair put it, “organizations exist only in so far as their members create them through discourse,” with discourse being “the principal means by which organization members create a coherent social reality that frames their sense of who they are.”Mumby, D. K., & Clair, R. P. (1997). Organizational discourse. In T. A. van Dijk (Ed.), Discourse as structure and process, Vol. 2 (pp. 181-205). London: Sage; pg. 181. In other words, communication is not just one activity, among many others, that an organization “does.” Rather, the organization itself is constituted through its members’ communication; it does not exist objectively, but only in relation to its members’ points of view.

This explains the ontology of interpretive theorists, their belief in how organizations have being. Their epistemology, or how these theorists believe knowledge is gained, is expressed by the word “interpretive.” Recall that postpositive theorists believe the mindsets of individuals do not matter since they are irresistibly forced choose the most effective courses of action; thus, to know an organization it is sufficient to observe its aggregate behaviors. By contrast, interpretive theorists believe that simple observation is insufficient; the mindsets of organization members must also be interpreted. Hence, this approach to studying organizational communication is called interpretivism. (Some theorists also use the term “social constructionism” to emphasize how social phenomena, such as organizations, are constructed through social interaction.)

But how do you interpret what is going on inside someone’s mind? Many methods are used. These usually begin by collecting primary data—interviews with people at various levels of the organization, and copies of organizational documents such as mission statements, annual reports, policy manuals, internal memoranda, and the like. Researchers who engage in organizational ethnographyThe word literally means “writing the culture.” Organizational ethnographers conduct fieldwork, perhaps spending a year or more to directly experience an organization through participation and observation. The goal is to describe the organizational culture in terms that are faithful its members’ understandings. do fieldwork in which they may spend a year or more visiting an organization, attending weekly staff meetings, participating in rituals such as office parties and company picnics, joining in ordinary conversations around the water cooler, and then recording their observations. Techniques for analyzing these ideographic data are also varied and include discourse analysis, conversation analysis, genre analysis, rhetorical analysis, and other methods.

These analytical methods involve an examination of how organization members use language to construct a shared social reality. By interpreting how language is used (e.g., company slang, recurring phrases, common metaphors, use of active and passive voice, what arguments employees find persuasive, how people address one another, how people take turns in talking) researchers uncover the underlying assumptions within an organization that its people take for granted and may not explicitly verbalize. Interpretive researchers, then, believe that organizational communication is not merely an instrument for getting results. Rather, people in organizations communicate with each other to make sense of their workplace and negotiate their places within the organization. Though managers may believe that precise instructions maximize productivity, directives that ignore employee perceptions can be disregarded, misinterpreted, and even counterproductive.

The axiology of interpretive scholars is evident from their research. Where postpositive researchers do not regard organization members’ individual mindsets (which cannot be directly observed or measured) as worth knowing, interpretive researchers believe these data and their interpretation are essential to understanding organizational life. Moreover, where the goal of postpositive researchers is to move from description and explanation to prediction of organizational behaviors, interpretive scholars believe that studying an organization on its own terms means producing a description and interpretation of organizational life that is faithful to its members’ own understandings. Interpretive researchers can, and do, make their findings about an organization’s communication and culture available to their subjects; in turn, organizations may use this information to address negative perceptions and to change a dysfunctional company culture into a more humane one. Interpretive researchers see their role not as changing the status quo but describing it. Yet identifying the unspoken assumptions that circulate within an organization may be the first step in addressing inhumane practices.

A summary of our discussion about the interpretive approach to organizational communication is presented in Table 4.2 "The Interpretive Approach" below.

Table 4.2 The Interpretive Approach

Ontology (how things exist) Epistemology (how things are known) Axiology (values for research) Purpose of Org. Communication
Relativism Interpretation Description Negotiation
Organizations come into existence and are then maintained through their members’ communication. Thus, organizations exist in relation to its members’ points of view. To learn about an organization, simple observation of aggregate behaviors is insufficient. The mindsets of members must also be interpreted. Research aims to describe the organization on its members’ own terms, although knowledge can be used to develop general theories and applied to management practices. People in an organization use communication to make sense of the work environment, establish shared patterns, negotiate their own identities, and enact their roles.

Critical Approach

A generation ago you might have read a company manual that stated, “When an employee is late to work, he must report immediately to his supervisor.” Today we read that sentence and, right away, notice its use of sexist language. But at the time, it was common to use the masculine pronoun as an inclusive reference for both genders. For decades, even centuries, the practice was so widely accepted as natural and self-evident that people did not question this use of the masculine pronoun. In the 1960s, for example, the mission of the starship Enterprise in the Star Trek television series was “to boldly go where no man has gone before.” Not until a sequel series debuted in the late 1980s was the mission statement rephrased, “To boldly go where no one has gone before.” Living in the twenty-first century, we now wonder how an earlier generation could have accepted sexist language without question. Yet consider: Which of our own assumptions will someday seem “unenlightened” to our children and grandchildren?

Think of some things we take for granted about the workplace. If someone asked you who “owns” the company you work for, you would answer with the name of person who is the “owner” in a financial sense. It just seems natural and self-evident that the one who holds the purse strings is the owner—even though you too have a tangible stake in the company and help make its activities possible. And in a free enterprise economy, we take for granted the notion that increased profits benefit everyone. Even college students, before they enter the workforce, accept this premise. Most young people attend college to “make more money” by learning job skills which will fit them into the needs of moneymaking corporations. Therefore, as corporations and their employees all make more money, everyone wins.

These assumptions illustrate what critical theorists call the reification and universalizationAccording to critical theory, the process by which dominant interests are represented as identical to everyone’s interests. Thus, to speak of “company interests” is, in reality, to speak of managerial interests. of managerial interests. ReificationAccording to critical theory, the process by which something historical is made to seem natural. Thus, the dominant interests within an organization appear to be natural and self-evident. is the process by which something historical is made to seem natural. For example, what we call the profit motive did not always exist; it emerged under specific historical conditions as premodern feudal economies gave way to modern capitalist economies. But we have so reified the profit motive that its pursuit seems natural, normal, self-evident, and beyond questioning. This process of reification produces a “double move” by ensuring that managerial interests are considered the only legitimate interests, while simultaneously hiding the domination of those interests by making them seem perfectly natural. Thus, the interests of management are universalized and represented as identical to everyone’s interests. To speak of “company interests” is, in reality, to speak of managerial interests. Such distortion becomes, from a critical perspective, the very purpose of organizational communication—that is, the operation of dominant interests to create a “false” consensus between management and employees.

Critical theorists, like the postmodern theorists we will review below, see the organization as a created by larger forces of history and society. But unlike postmodern theorists who see the organization in constant flux within the swirling streams of those forces, critical theorists tend to see the structures of power and domination as being so reified that they constitute “a concrete, relatively fixed entity.”Deetz (2001), op. cit., pg. 27. Again, this requires a decision about ontology or the nature of existence—in our case, about the nature of organizational existence.

In addition to reification and universalization, critical theorists are concerned with two more questions: how reasoning in organizations becomes grounded in “what works,” and how dominant managerial interests gain the consent of subordinate interests. Jurgen Habermas noted how the modern age has increasingly supplanted practical reasoning that seeks mutual consensus, with technical reasoningReasoning that, according to cri+tical theory, calculates the means and controls needed to accomplish a desired end. In organizations, technical reasoning is made to seem that only rational basis for decisions. For the modern age it has largely replaced practical reasoning which seeks mutual consensus. that calculates the means and controls needed to accomplish a desired end.Habermas, J. (1971). Knowledge and human interests (J. Shapiro, Trans.). Boston: Beacon. Critical theorists have applied this insight to organizational life by critiquing how corporations make technical reasoning, or determination of “what works” in achieving managerial interests, appear to be the only rational approach. Practical reasoning that fosters mutual determination of organizational goals either is made to seem irrational or is even leveraged by management as another “technique” to further its own interests. Thus, ideas such as promoting worker participation are either dismissed as inefficient or used as a new means to bring workers into alignment with corporate interests. Why do workers consent to such domination? Critical theorists have looked at bureaucratic forms, at coercions and rewards, and at organizational cultures that provide no chance for alternative modes of thinking—or that cause employees to identify so completely with an organization, they internalize its goals into their sense of personal duty and job satisfaction. Such employees need not be controlled; they discipline themselves.

If, according to critical theorists’ ontology, organizational structures have been reified into an objective existence, then according to their epistemology, how do these taken-for-granted structures become known? Most critical organization researchers engage in ideology critiqueAn approach to critical scholarship that employs theory to expose how dominant interests distort meaning, thought, and consciousness to simulatenously legitimize and hide their domination.. These researchers bring to their subjects an existing theory and then use it as a framework to expose how a dominant ideology has operated to reify and universalize its interests. A good example is provided by Karl Marx, the originator of ideology critique. He theorized that differences between capital and labor are built into the very structure of the capitalist system and its ideology. Then he employed his theory to explain how the few (who owned capital) could not only exploit the many (who owned only their labor), but could also make their domination appear legitimate and natural. Notions of economic class differences remain an influential strand of ideology critique. Yet other bases for criticism have also become important. More recently, feminist theory offers another example of ideology critique as researchers bring theories about gender-based structures of domination and use these as frameworks to expose or “denaturalize” the patriarchal assumptions that organizations take for granted.

In addition to ideology critique, a second stream of critical scholarship has emerged that follows the theories of Jurgen Habermas about communicative actionAn approach to critical scholarship that examines how dominant interests distort communication processes to sustain their domination by foreclosing alternate expressions. But legitimate communication may be restored, it is argued, through greater democratization of the workplace..Habermas, J. (1984). The theory of communicative action: Vol 1., Reason and the rationalization of society (T. McCarthy, Trans.). Boston: Beacon; Habermas, J. (1987). The theory of communicative action: Vol. 2, Lifeworld and system. Boston: Beacon. Where critical scholars have traditionally plumbed the ways that meaning, thought and consciousness itself are distorted by dominant discourses, Habermas began in the late 1970s to explore how communication is distorted. He proposed that, ideally, a communicative act should satisfy four conditions: participants should have equal opportunities to speak, should be heard without preconceptions of what is “true” and “proper,” and should be able to speak according to their own lived experiences. Scholars, then, can critique how organizations distort these conditions. Thus, managers have more opportunities to speak; “bottom line” considerations are a privileged form of knowledge and seen as the only rational basis for resolving issues; the organization’s structure dictates, in advance, the proper relations between management and labor; and discussions of workplace concerns must take place within the context of “company” (i.e., managerial) interests. Yet Habermas’s model for communicative action also suggests possibilities for a positive agenda. A number of scholars have proposed how legitimate organizational communication can be restored through democratization of the workplace.For example, see Cheney, G. (1995). Democracy in the workplace: Theory and practice from the perspective of communication. Journal of Applied Communication Research, 23, 167-200; Deetz, S. (1992). Democracy in the age of corporate colonization: Developments in communication and the politics of everyday life. Albany: State University of New York Press; Harrison, T. (1994). Communication and interdependence in democratic organizations. In S. Deetz (Ed.), Communication Yearbook 17 (pp. 247-274). Thousand Oaks, CA: Sage.

As we learned above, postpositive and interpretive theorists look for order to emerge in organizations. Postpositive researchers look for the ways that organizational imperatives for efficiency and productivity brings members into alignment; interpretive researchers look for the ways that members create, through their communication, stable communities and shared cultures. By contrast, critical theorists believe that organizations are sites where historical and societal ideologies are in conflict, and where reified structures produce dominant and subordinate discourses. Other researchers may look for surface stability, but critical theorists’ axiology regards an organization’s submerged voices—workers, women, people of color—as worth knowing. Critical theorists’ scholarship aims to recover these marginalized voices, lay bare an organization’s reified structures for all to see, reopen possibilities previously foreclosed by those structures, and replace false consensus with true consensus. Given that emancipation is their goal, critical researchers combine scholarship with activism. These qualities—the ontology, epistemology, and axiology of the critical approach to organizations—are summarized in Table 4.3 "The Critical Approach" below.

Table 4.3 The Critical Approach

Ontology (how things exist) Epistemology (how things are known) Axiology (values for research) Purpose of Org. Communication
Realism Critique Emancipation Distortion
The power structures of organizations have an objective existence formed by external historical and cultural forces and that is independent of the people in them. Exposing the hidden power structures in organizations is accomplished by using general theories about oppression as a framework to analyze a particular organization. Research aims to expose the hidden power structures in organizations so that marginalized interests can resist and previously foreclosed opportunities become possible. Communication by the dominant interests in organizations systemically distorts meaning, thoughts, consciousness, and communicative actions so that domination seems natural and alternative expressions are foreclosed.

Postmodern Approach

Many critical theorists hold that historical and cultural forces produce power structures with fixed existences, but postmodern theorists of organizational communication take a different view. “Reality” constantly fluctuates in the ongoing contests among competing historical and cultural discourses. Humans themselves are sites of competition between these discourses so that—despite our conceit that we have autonomous identities and control our own intentions—we are products of the multiple voices which shape and condition us. As Robert Cooper and Gibson Burrell explained, postmodern theorists “analyze social life in terms of paradox and indeterminacy, thus rejecting the human agent as the center of rational control and understanding.” In contrast to the modernist approach in which the “organization is viewed as a social tool and an extension of human rationality,” the postmodern approach sees the “organization [as] less the expression of planned thought and calculative action and more a defensive reaction to forces extrinsic to the social body which constantly threaten the stability of organized life.”Cooper, R., & Burrell, G. (1988). Modernism, postmodernism, and organizational analysis: An introduction. Organization Studies, 9, 91-112; pg. 91. Perhaps an analogy can help. Imagine the sweep of great historical and cultural discourses as an ocean. Its constantly swirling waves and currents determine the actions and perceptions of all who sail upon it. An organization, then, is like a flotilla of ships that negotiate a temporary agreement to sail as a convoy until land is reached. Though an organization may last for decades, rather than days or weeks, in time the currents of history and society that brought it together will pull it apart. Postmodern theorists therefore reject the notion that, as a social object, the “organization” has an objective and enduring existence.

As postmodern theories of organizational communication have developed over the past generation, several themes have emerged. First is the centrality of discourseBecause postmodernists believe that language is the decisive factor in constructing societies, organizations and individuals, then discourse is the central focus of their studies., so that an organization is regarded as a “text” that postmodern analysts can “read.” The goal in “reading” this “text” is to unravel the underlying—and hidden—historical and cultural discourses which are reflected in the organization. This focus on discourse also means that postmodern scholars view language, rather than thought or consciousness, as the decisive factor in the social construction of organizations. Individuals are not the bearers of meaning, but are caught in webs of meaning that language creates. Following on this idea, a second theme emerges. Organizations are said to be de-centeredBecause postmodernists believe that individuals are not autonomous but are shaped by language, they hold that individual free will is not the central driving force of an organization.; the free will of its members are not the central driving force since people are preconditioned by language. Moreover, since organizations are only clusters of temporary consensus between competing discourses, then over time theses discourses tend to produce fragility rather than unity. Different levels of organization—from executives and middle managers to office employees and field personnel—look at things according to their own experiences and interests. Their multiple voices generate varying perspectives, producing multiple social realities rather than a single organizational culture. Thus, postmodern scholars say that organizations are fragmentedBecause postmodernists believe that organizations are temporary and fluid combinations of differing interests, they hold that the various discourses of the interests do not produce the stability of a single unified pattern but instead generate multiple social realities that lead to organizational fragility and fragmentation.. Nor is clash of voices without effect on individuals, who are shaped by the multiple discourses operating throughout the organization and surrounding society. For this reason, postmodernists say that individuals’ organizational identities are overdeterminedBecause postmodernists believe that individuals are not autonomous but are sites where multiple discourses are simultaneously in conflict, then identities of people within organizations are always fluid and partial—and thus overdetermined—rather than stable and continuous. and therefore precarious and unstable.

Given that each organization is a unique “text” that is ever fluid, a third theme in postmodern analysis is what Jean-Francois Lyotard called an “incredulity toward metanarratives.”Lyotard, J.-F. (1984). The postmodern condition: A report on knowledge (G. Bennington & B. Massumi, Trans.). Minneapolis: University of Minnesota Press; pg. xxiv. In contrast to critical scholars who look at organizations through the prism of an overarching theory—such as Marx’s theories about class struggle or Habermas’s theories about communicative action—postmodern scholars deconstruct the “text” of an organization for what it is, without fitting it into an a priori theoretical framework or metanarrativePostmodernists regard with incredulity that suggestion that a single “great story,” such as an overarching general theory, can provide all the answers.. Thus, a fourth theme in the postmodern approach to organizations is the need to deconstructPostmodernists believe that an organization is a “text” that can be “read.” Deconstruction is the method by which analysts trace back the discourses that have formed the power relations within an organization. the particular connections, within each organization, between knowledge and power. What we call organizational communication is, for postmodernists, the ongoing contest between discourses. The dominant interest works to sustain its power by ensuring that organizational knowledge is rendered on its own terms and other interpretations seem unnatural. Postmodern scholars strive to reopen taken-for-granted discourses of knowledge and power, trace their formation, and help recover the voices which have been marginalized.

In tracing out the knowledge/power connection, many postmodern organizational scholars follow the work of French philosopher Michel Foucault.See Foucault, M. (1977). Discipline and punish: The birth of the prison A. Sheridan, Trans.). New York: Pantheon; Foucault, M. (1980). The history of sexuality (R. Hurley, Trans.). New York: Pantheon; Foucault, M. (1988). Technologies of the self. In L. H. Martin, H. Gutman & P. H. Hutton (Eds.), Technologies of the self: A seminar with Michel Foucault (pp. 16-49). Amherst: University of Massachusetts Press. Like Foucault, these scholars are concerned with the ways that modern organizations have eliminated the need to enforce discipline through physical punishment and real-time surveillance, but have “manufactured consent” and thereby “produced” employees who willingly discipline themselves. Nevertheless, Foucault did not see power as all bad. He held that power relations, being “rooted deep in the social nexus,” are inescapable; they arise from the fact of society itself and are therefore not “a supplementary structure whose radical effacement one could perhaps dream of.” The goal is not to abolish power and somehow create a perfectly free society, for a “society without power relations can only be an abstraction.” Rather, the goal is a more nuanced understanding that makes possible “the analysis of power relations in a given society, their historical formation, the source of their strength or fragility, the conditions which are necessary to transform some or to abolish others.” Toward that end, power may be seen not only as constraint; it also enables action by marking out ranges of possibilities and channels for their realization. Postmodern scholarship lays bare the power relations within organizations, putting these relations back into play and helping marginalized voices restructure the field of action to open up previously foreclosed possibilities.Foucault, M. (1982). The subject and power. Critical Inquiry, 8, 777-795; pg. 791.

For postmodernists, then, the three decisions that organizational theorists must make—about ontology (how things exist), epistemology (how things are known), and axiology (what is worth knowing)—are summarized in Table 4.4 "The Postmodern Approach" below.

Table 4.4 The Postmodern Approach

Ontology (how things exist) Epistemology (how things are known) Axiology (values for research) Purpose of Org. Communication
Relativism Deconstruction Denaturalization Contestation
Organizations come into existence as temporary combinations of interests against the threatening fluidity of larger historical and cultural discourses, so that they exist only in relation to those external forces. Organizations are “texts” that can be “read.” The goal is to deconstruct, or trace back, the historical and cultural discourses that led to the formation of a particular organization’s power relations. In the ongoing contest between organizational discourses, dominant interests maintain power by ensuring organizational knowledge is rendered on its own terms and made to seem natural. Research aims to “denaturalize” and thus reopen hidden power relations. Organizational communication is a means by which the discourses of an organization’s various interests are contested. In this contest, some discourses dominate and others are marginalized.

Combining Approaches

Until the 1970s, organizational research mostly proceeded from what is now called a postpositive approach. Articulating new paradigms, Stanley Deetz noted, “gave legitimacy to fundamentally different research programs and enabled the development of different criteria for the evaluation of research.”Deetz (2001), op. cit., pg. 8. At the same time, however, labeling has created distinctive communities of researchers that each favor a particular paradigm and can sometimes ignore or even dismiss the work of others.

The authors of this textbook individually take different approaches to organizational communication research. Yet we believe all perspectives make valuable contributions. For example, we share a common interest in communication by members of organized religions. Jason Wrench and Narissra Punyanunt-Carter have conducted, with other colleagues, extensive surveys of religious believers to produce an aggregate statistical picture of their communication behaviors.Punyanunt-Carter, N. M., Corrigan, M.W., Wrench, J. S., & McCroskey, J. C. (2010). A quantitative analysis of political affiliation, religiosity, and religious-based communication. Journal of Communication and Religion, 33, 1-32; Punyanunt-Carter, N. M., Wrench, J. S., Corrigan, M. W., & McCroskey, J. C. (2008). An examination of reliability and validity of the Religious Communication Apprehension Scale. Journal of Intercultural Communication Research, 37, 1-15; Wrench, J. S., Corrigan, M. W., McCroskey, J. C., & Punyanunt-Carter, N. M. (2006). Religious fundamentalism and intercultural communication: The relationship among ethnocentrism, intercultural communication apprehension, religious fundamentalism, homonegativity, and tolerance for religious disagreements. Journal of Intercultural Communication Research, 35, 23-44. By contrast, Mark Ward spent several years visiting the local churches of a religious sect, participating in their worship and rituals, observing their communication firsthand, and learning how they talk among themselves.Ward, M., Sr. (2009). Fundamentalist differences: Using ethnography of rhetoric (EOR) to analyze a community of practice. Intercultural Communication Studies, 18, 1-20; Ward, M., Sr. (2010). “I was saved at an early age”: An ethnography of fundamentalist speech and cultural performance. Journal of Communication and Religion, 33, 108-144; Ward, M., Sr. (2011). God’s voice in organizational communication: A root-metaphor analysis of fundamentalist Christian organizing. Paper presented at National Communication Association 97th Annual Convention, New Orleans, LA, November 2011; Ward, M., Sr. (in press). Managing the anxiety and uncertainty of religious otherness: Interfaith dialogue as a problem of intercultural communication. In D. S. Brown (Ed.), Interfaith dialogue: Listening to communication theory. Lanham, MD: Lexington. To use Deetz’s distinction cited above, Ward derives “knowledge from” insiders on their own terms, while Wrench and Punyanunt-Carter consult existing theories and apply that “knowledge to” observed behaviors. Yet we see these different approaches not as an “either/or” choice but as complementary. “Insider” research contributes detailed cases of organizational communication that, taken together with cases of other organizations, may help construct more robust general theories. On the other hand, theoretically-informed research may identify broad patterns in organizational communication that can help those doing “insider” research make sense out of hundreds of separately collected observations. Figure 4.5 "Research Approaches as Complementary" below provides a graphic representation of this dynamic.

Figure 4.5 Research Approaches as Complementary

Key Takeaways

  • Different conceptions of an “organization” are behind different approaches to theorizing about organizational communication. The postpositive approach holds that an organization has an objective existence; people come and go, but the organization endures. The interpretive approach holds that an organization has a subjective existence; people create and sustain it through their communication. The critical approach holds that the structures of power within an organization have a fixed existence and reflect larger historical and cultural forces. The postmodern approach also holds that the power relations within an organization reflect larger historical and cultural discourses, but that these discourses are fluid and ever changing.
  • Theories of organizational communication reflect assumptions with regard to ontology (how things, including social phenomena such as organizations, have existence), epistemology (how things become known), and axiology (what is worth knowing). In the previous takeaway, the ontologies of the four approaches to theorizing about organizational communication are described. The postpositive approach holds that organizations are known through scientific inquiry and that only empirical findings are worth knowing. The interpretive approach holds that organizations are known by directly experiencing them and that individuals’ perceptions (though these cannot be measured) are worth knowing. The critical approach holds that hidden power structures are exposed by applying general theories about domination and that the voices of marginalized groups are worth knowing. The postmodern approach also holds that marginalized discourses are worth knowing, but that hidden power relations are exposed by deconstructing, or tracing back, how various discourses have formed in a given organization.

Exercises

  1. Think of the college or university that you are attending. Then imagine a prospective student asking you, “What is the best way to find out what your school is really like?” Would you advise the prospect to take a survey of current students, or to spend some time living on campus and participating in school activities? What is the reason for your advice? Could you imagine how a combination of both methods could be useful? Explain yourself.
  2. We described above how most students go to college to “make more money,” taking for granted that higher education is about fitting into the needs of corporations. Can you think of other ways that the corporate world has influenced college students so that you might think in ways that serve corporate interests? Why might these thoughts seem natural to you, so that you do not question them?
  3. Which approach to theorizing organizational communication—postpositive, interpretive, critical, or postmodern—makes the most sense to you? Why? Explain your answer.

4.2 Rethinking Communication

Learning Objectives

  1. Differentiate among the three models for how communication functions—linear, interactional, and transactional—and the limitations of each model.
  2. Differentiate among the seven traditions of communication theory and understand how each approaches the nature of communication and how meaning is exchanged.

You were introduced in Chapter 1 "Introduction to Organizational Communication" to the “SMCR” model of communication. For two good reasons, numerous textbooks in communication begin with this longstanding model. First, its components—source, message, channel, receiver—are easy to grasp. In our modern world of phones, computers, networks and mass media, we encounter the basic idea of the SMCR model on a daily basis. And so, second, the model is effective in getting students to think—often for the first time—about “communication” as more than just a reflex action, more than something that just “happens.”

In this section we will consider the two questions: how communication works and what communication is. The SMCR model, for example, suggests communication works by traveling in a straight line from source to receiver. But scholars have largely moved beyond this simple linearA model of communication which holds that a message travels in a straight line from its source, through a channel, and to its receiver. model and have described communication as an interactionalA model of communication which holds that communication travels in a circle as a sender transmits a message and then the receiver responds with feedback; thus both parties become sender/receivers. or, more recently, a transactionalA model of communication which holds that sending and receiving of messages/feedback occurs simultaneously. process. Below, we will review these three models below of how communication works. Yet an even more basic question concerns what communication is. The SMCR model belongs to a body of theories that conceive of communication as information processing, an approach that is called (as we will explain below) a “cybernetic” concept of communication. Yet the cybernetic concept is not the only body of communication theories. As Robert Craig described, seven distinct traditions of communication theory have emerged.Craig, R. T. (1999). Communication theory as a field. Communication Theory, 9, 118-160. Since modern theories of organizational communication are often built on a different concept of communication than a cybernetic one, then later in this section we will review the seven approaches to answering the question: What is communication?

How Communication Works: Three Models

At the most basic level, the three models of how communication works—linear, interactional, and transactional—can be represented by the three graphics in Figure 4.6 "Three Concepts of Communication" below. The linear model originated in the 1940s, the interactional in the 1950s, and the

Figure 4.6 Three Concepts of Communication

transactional in the 1970s. That the original linear model of communication remains influential is attested by its inclusion in so many introductory textbooks—including this one. But theorists have long noted its limitations: the assumptions that listeners are passive, that only one message is transmitted at a time, that communication has a beginning and an end. In fact, a source could transmit a confusing or nonsensical message, rather than a meaningful one, and the linear model would work just as well; there is no provision for gauging whether a message has been understood by its receivers. Neither is the context of a communication situation taken into account. Nevertheless, the linear model introduces helpful concepts and terms that are the basis for understanding, as we will see later, the interactional and transactional models of communication.

Linear Model

Inspired by postwar research at Bell Laboratories on telephone transmissions, Claude Shannon and Warren Weaver developed the “mathematical model” of human communication shown in Figure 4.7 "Linear Model of Communication: Shannon & Weaver" below.Shannon, C., & Weaver, W. (1949) The mathematical theory of communication. Urbana: University of Illinois Press, 1949. In their model, successful sending and receiving of a message is a function of the channel’s capacity to handle signal degradation caused by static noise on the line. When applied in general to human communication, “noise” can be physical (background noises that make the message harder to hear), physiological (impairments such as hardness of hearing), semantic (difficulties in understanding choices of words), and psychological (predispositions and prejudices that affect how the message is interpreted). As you can see in Figure 4.7 "Linear Model of Communication: Shannon & Weaver", communication travels in a straight line.

Figure 4.7 Linear Model of Communication: Shannon & Weaver

A decade after Shannon and Weaver, David Berlo adapted their concepts into the now-familiar SMCR (source, message, channel, receiver) model.Berlo, D. (1960). The process of communication. New York: Holt, Rinehart, and Winston. This is the model we introduced in Chapter 1 "Introduction to Organizational Communication" and have reproduced in Figure 4.8 "Linear Model of Communication: Berlo" below. Berlo’s adaptation was “tremendously influential” in offering a more flexible and “humanized conception of Claude Shannon’s model” that facilitated its application to oral, written, and electronic communication.Rogers, E. M. (2001). The department of communication at Michigan State University as a seed institution for communication study. Communication Studies, 52, 234-248; pg. 234. Moreover, the notion of feedback provided a means for gauging reception and understanding of the message. Yet as we will see below in the descriptions of the interactional and transactional models, subsequent theorists have attempted to show how communication is better understood as circular rather than linear, how listeners are also active participants in communication, how multiple messages may be sent simultaneously, and how context and culture impact understanding.

Figure 4.8 Linear Model of Communication: Berlo

Interactional Model

Only a few years after Shannon and Weaver published their one-way linear model, Wilbur Schramm proposed an alternate model that portrayed communication as a two-way interaction.Schramm, W. (1954). How communication works. In W. Schramm (Ed.), The process and effects of communication (pp. 3-26). Urbana: University of Illinois Press. Writing several years before Berlo, he was the first to incorporate feedback—verbal and nonverbal—into a model of communication. The other important innovations in Schramm’s interactive model, which we have adapted in Figure 4.9 "Interactional Model of Communication" below, were the additions of the communication context (the specific setting that may affect meaning) and of “fields of experience” (the frames of reference and the cultures that each participant brings to the communication).

Figure 4.9 Interactional Model of Communication

With Schramm’s model, communication moves from a linear to a circular process in which participants are both senders and receivers of messages. Yet the model portrays communication like a tennis match: one participant serves up a message and the other participants then makes a return. Each waits, in turn, passively for the other. Thus, communication goes back and forth as one person (on the left of Figure 4.7 "Linear Model of Communication: Shannon & Weaver") initiates a message and waits until the other (on the right) responds. But if you think about times when you have engaged in conversation, you will recognize how the other person is simultaneously sending messages—often nonverbally—while you are talking. Unlike a tennis match, you do not wait passively until the “ball is in your court” before acting communicatively. To demonstrate the simultaneity of communication, we move next to a transactional model.

Transactional Model

Perhaps the first model to portray communication as a simultaneous transaction is attributed to Dean Barnlund.Barnlund, D. (1970). A transactional model of communication. In K. K. Sereno & C. D. Mortensen (Eds.), Foundations of communication theory (pp. 83-102). New York: Harper. Later theorists have developed this idea of simultaneity, which is illustrated in Figure 4.10 "Transactional Model for Communication" below. As you can see, messages and feedback are being exchanged at the same time between communicators. And because they are engaged together in the transaction, their fields of experience overlap. Useful concepts such as noise and context can likewise be added to the model.

Figure 4.10 Transactional Model for Communication

An expanded view of how communication functions can help us to better understand how individuals within organizations communicate. But for a firmer grip on modern theories of organizational communication we will now go beyond the message-centered, functional models described above and take a meaning-centered approach.

What Communication Is: Seven Traditions

You have probably heard the proverbial question: If a tree falls in the forest and no one is around to hear, does it make a sound? Similarly, we might ask: If you send a message that the receiver does not understand, has communication taken place? This question introduces the idea of meaning into the equation. Let us borrow from the SMCR model one more time to explore the place of meaning in communication.

Some theorists believe (as you probably do) that the meaning of a message lies in the sender. You think up a message and transmit it, and then the receiver must decode what you mean. But other theorists believe the meaning of a message is something that the sender and receiver construct together as they interact through their communication. Still other theorists believe that meaning resides in the channel—perhaps in the signs and symbols that, over time, humans invest with implied meanings, or perhaps in the larger structures of history and culture that condition how we perceive the world. As noted at the start of this section, Craig has identified seven traditions—which are summarized in Table 4.5 "Seven Traditions of Communication Theory" below—in communication theory.Craig, op. cit. Each wrestles with the question of how people derive meaning from a communication. And if we grant that communication only takes place when meaning is exchanged, then the issue of how people derive meaning is another way of putting the question: What is communication?

A helpful way of grasping the seven theoretical traditions is to pose a single communication scenario and then consider it from each of the seven approaches. For our purposes, we will pick a common scenario from organizational life—namely, the annual employee recognition luncheon in which awards are given to those who reach five or ten or fifteeen years of service, and so on, up until retirement. During this festive event a catered lunch is served in a large room, speeches are made by key executives, long-serving employees come forward as their names are called and receive a certificate or plaque, and the luncheon concludes on a light note as employees organize a mock ceremony to give out humorous awards. For our overview of the seven traditions, let us begin with the tradition to which you have already been introduced—the cybernetic tradtion—and see how it might explain our communication scenario.

Table 4.5 Seven Traditions of Communication Theory

Theoretical Tradition Communication theorized as...
Cybernetic information processing
Phenomenological experience of otherness
Sociopsychological expression, interaction, influence
Sociocultural (re)production of social order
Semiotic intersubjective mediation by signs
Critical discursive reflection
Rhetorical practical art of discourse

Cybernetic Tradition

Theorists in the cyberneticA scholarly tradition that theorizes communication as information processing. tradition start with the assumption that an organization is a system comprised of many interdependent parts. The annual employee recognition luncheon is a particularly good occasion to see all those parts in action:

  • The top executives who make speeches and set policies for giving awards;
  • The managers who implement the policies;
  • The human resources department that generated the list of employees eligible for awards and organized the luncheon;
  • The corporate communications department that will send out a press release after the event;
  • The accounting department that processed purchase orders and payments to the caterer;
  • The information technology department that set up the audiovisual equipment for the awards ceremony;
  • The maintenance department that prepared the room and will clean up afterward; and finally,
  • The employees who attended the luncheon, received awards, and put on the humorous entertainment.

All of these parts depend on each other—and must communicate together—to make the annual employee recognition luncheon happen. In the cybernetic tradition, then, communication is theorized as information processing. But cybernetic theorists do not stop at charting information pathways. They are also interested in how a system continually makes adjustments needed to sustain itself. Indeed, the word “cybernetics” was coined from the Greek word for “steersman” by MIT scientist Norbert Wiener.Wiener, N. (1954). The human use of human beings: Cybernetics in society Boston: Houghton Mifflin. In devising a new antiaircraft firing system during World War II, he addressed a major problem: though existing systems could feed back information on firing trajectories, targets would pass by before human operators could make adjustments. He saw that the new system must regulate itself by acting on its own feedback, a principle Wiener then extended to human societies. Communication theorists picked up on this idea by casting the communication process as a self-regulating system in which people act on feedback, adjust their messages, gradually eliminate distortions, and arrive at intended meanings.

Adjustments are made via feedback loops which connect the various parts of the system into networks. Our example of the employee awards luncheon illustrates several of these networks in play. Top executives, who want to annually honor loyal employees, must get feedback from the human resources department for a list of who is eligible. To organize the event, the human resources department must get feedback from the maintenance department on the room setup, the IT department on audiovisual equipment, and the accounting department on the budget for the caterer. To publicize the event, the corporate communication department must get feedback from top executives on the desired tone or theme of the press release. Moreover, the system cannot survive just by feeding on itself. Inputs and resources are gathered from the surrounding environment—for example, by soliciting proposals from local caterers, and by talking to local media about possible news and feature story angles. Through all these avenues of organizational communication, the system processes the information it needs to keep on going.

Phenomenological Tradition

Imagine yourself as a new employee who is attending the annual recognition luncheon for the first time. As you watch the first group of honorees go forward and accept their five-year service certificates, you picture yourself in their shoes and ponder, “Is this company a place I want to be in five years? Or is it a stepping stone?” Then you see the ten-year honorees and think, “Wow, ten years! If I’m still here in ten years, that means I’m committed long-term.” Also, you notice that ten-year employees tend to be people who have better job titles and higher pay, so that longevity has its rewards. Finally, you see plaques handed out to retirees and say to yourself, “I can’t even relate! What will my career have been like when I look back on it, someday? What do I want to be known for?” In the days after the luncheon, you run into some five- and ten-year honorees you know, tactfully engage them in conversation, and try to feel out their answers to the question, “Is it worth it to stay long enough to earn a service award?”

According to the phenomenologicalA scholarly tradition that theorizes communication as dialogue and the experience of otherness. tradition of communication theory, you derive meaning by directly experiencing a particular phenomenon. At the luncheon you are confronted with the phenomenon of employee loyalty and longevity, and based on this experience you weigh your perceptions. Thus, you come to know your organizational world by directly and consciously engaging in it, pondering its meaning for you, interpreting that meaning through language to define and express it, and then continually reconstructing the interpretation in light of new experiences. Dialogue is another important concept in the phenomenological tradition. The annual luncheon was a type of dialogue as you listened to the various speeches and presentations. Then after the event, you dialogued one-on-one with coworkers who had been honored for their long service. Through these dialogues you open yourself to the experiences of others and can integrate this into your own experience.

Sociopsychological Tradition

In Chapter 1 "Introduction to Organizational Communication" you were introduced to a definition of human communication as a “process whereby one individual (or group of individuals) attempts to stimulate meaning in the mind of another individual (or group of individuals) through intentional use of verbal, nonverbal, and/or mediated messages.” We offered this definition in the opening chapter because it is a good place start. For one, the definition is held by many communication theorists. For another, it accords with what most laypeople (probably including you) believe about communication and about personhood. You likely see yourself as a distinct individual; your mind is your own. This is the basic assumption of sociopsychologicalA scholarly tradition that theorizes communicaiton as expression as expression, interaction, and influence rooted in human psychological processes. theories about communication, that people control their own intentions. Thus, as noted above, communication may seen as one person’s intention to impact another person’s intention. Such a notion is problematic, however, for many communication theorists. Where sociopsychological theorists see individuality as an objective fact, postmodern theorists hold that people’s intentions are subjectively conditioned by their histories and societies. And where sociopsychological theorists believe that the meaning of a communication resides in the individual, sociocultural theorists (as will review below) believe that meaning arises from the interaction.

But for now, let us follow the sociopsychological tradition and see how it might explain the annual employee recognition luncheon. First, consider the speeches given by top executives to celebrate company values and, by implication, the loyalty these values merit from employees. One theory suggests that, psychologically, you are more likely to be persuaded if sufficiently motivated to carefully consider the arguments, and less likely if the speakers utter cliches you’ve heard before.Petty, R. E., & Cacioppo, J. T. (1986). Communication and persuasion: Central and peripheral routes to attitude change. New York: Springer-Verlag. Another theory claims that opinions are best understood not as a single point on a line, but as a continuum between acceptable and unacceptable; the more that the execs pitch their arguments on company loyalty toward the edge of this continuum, the more likely they can push the boundaries of what you will accept.Sherif, M., Sherif, C., & Nebergall, R. (1965). Attitude and attitude change: The socialjudgement-involvement approach. Philadelphia: Saunders. Still another leading theory proposes that if the speakers can make you feel an inner conflict between self-interest and group loyalties, you will be psychologically driven to resolve the conflict rather than feel torn.Festinger, L. (1957). A theory of cognitive dissonance. Stanford, CA: Stanford University Press.

Then there are the conversations you had with longer-tenured coworkers. One theory of interpersonal communication holds that people’s personalities are structured like the layers of an onion; to elicit your coworkers’ inner feelings about staying long-term with the company, you had to go beyond mere chit-chat about sports and the weather, and instead penetrate into their goals, convictions, fears, fantasies and, at the deepest level, their self-concepts.Altman, I., & Taylor, D. A. (1973). Social penetration: The development of interpersonal relationships. New York: Holt, Rinehart, and Winston. Another theory claims that people experience an ongoing psychological tension between their need for being connected and need for feeling unique, and between their need for being open and need for keeping some things to themselves.Baxter, L. A., & Montgomery, B. M. (1998). A guide to dialectical approaches to studying personal relationships. In B. M. Montgomery & L. A. Baxter (Eds.), Dialectical approaches to studying personal relationships (pp. 1–15). Mahwah, NJ: Erlbaum. In order to elicit coworkers’ true feelings about their service with the company—and to expose your own concerns—you must both navigate these tensions.

The main lesson here is that a sociopsychological view locates the meaning of communication within the mind of each individual. The company executives acted with the intention of promoting employee loyalty in the hope of influencing your intention. And when you acted on your intention to elicit information from long-serving coworkers, they were prompted by their own intention to be more, or to be less, open toward your questions. Human communicative behaviors, then, are seen as seen as rooted in human psychologies. So, if communication is defined as a process whereby one person intends to stimulate meaning in the mind of another, then the task of the researcher is to discover what stimuli elicit what responses.

Sociocultural Tradition

For the sociopsychological theorist, the meaning of a communication resides in each individual. But for the socioculturalA scholarly tradition that theorizes communication as the production and reproduction of a social order, such as an organizational culture. theorist, the meaning of a communication arises from interaction as people engage in discourse and socially construct what they jointly perceive to be real. George Herbert Mead, a founder of the sociocultural tradition, noted more than a century ago that—in contrast to the prevailing view that each individual is autonomous—people only develop a sense of self by being around other people. Further, since speech is the means by which people interact, then people develop their sense of self through communication. Indeed, without language—which arose because humans exist in society—there would be no thought.Mead, G. H. (1934). Mind, self, and society. Chicago: University of Chicago Press. Another theorist in the sociocultural tradition, Erving Goffman, likened social interaction to a drama. Imagine yourself in an ordinary conversation and (being honest) think how you take a role (anything from clown to peacemaker) and “play to the audience” by communicating in ways that (you believe) will make you socially acceptable.Goffman, E. (1959). The presentation of self in everyday life. Garden City, NY: Doubleday; Goffman, E. (1971). Relations in public. New York: Basic. W. Barnett Pearce and Vernon Cronen have described conversational interaction as a “coordinated management of meaning” in which people not only co-construct a social world but are, in turn, shaped by that world.Pearce, W. B., & Cronen, V. (1980). Communication, action, and meaning. New York: Praeger.

Given these assumptions, theorists in the sociocultural tradition look at the ways communication is used by people in interactions to produce—and then reproduce—stable patterns of social order. Sociocultural theorists of organizational communication, then, are interested in how organizational cultures arise as their members communicate with one another. And they would take a keen interest in the annual employee awards luncheon. First, there is the ritual aspect of the event as people on the platform speak structured sequences of words (an employee’s name is called, he or she comes forward, and the certificate is given with praise, smiles, and handshakes) that ultimately pay homage to the sacred object of the company. Second, the awards ceremony constitutes a story which fosters a “loyalty myth.” As the myth is enacted, the audience learns how they too are expected to fit into the story. Then, third, the awards ceremony is a “social drama” in which awardees gain honor by their perseverance, thus showing the audience how they can likewise win approval and continue to belong.

Organizational cultures are maintained not only through public events but also in natural conversation as employees spontaneously use “insider” talk. Such talk begins to form patterns that reproduce the values and assumptions of an organization culture.Philipsen, G. (1997). A theory of speech codes. In G. Philipsen and T. L. Albrecht (Eds.), Developing communication theories (pp. 119–156). Albany: State University of New York Press. Over time, the patterns seem so natural that employees use the talk without thinking and take the underlying cultural assumptions for granted. For example, if people address each other with formal titles—or, alternately, if they use first names—this talk reproduces assumptions about how organization members should relate to one another. Sociocultural researchers often look for words and phrases that keep recurring in significant ways. So perhaps the employee awards luncheon featured talk about the company as a “family” (a metaphor), or praised award recipients for being “customer-oriented” (a stock phrase), or continually referred to “aggressive” growth, “aggressive” marketing, an “aggressive” strategy, and so forth (a buzzword). Chances are that, when you later spoke one-on-one with award recipients, their use of such insider language in spontaneous conversation reflected their integration into the organizational culture.

Semiotic Tradition

The old saying, “Where there’s smoke, there’s fire,” captures the essence of the semioticA scholarly tradition that theorizes communication as intersubjective mediation by signs, or the ways a sign (including a word) or symbol of a thing mediates the different thoughts that people have about the thing and thus permit meaning to be shared. tradition in communication theory. Semiotics is the stidy of signs—and a classic example, of course, is how the presence smoke is the sign of a fire. Charles Saunders Peirce, a founding theorist of semiotics, would have called smoke an index or a trace that points to another object.Peirce, C. S. (1958). Charles S. Peirce: Selected writings (P. P. Weiner, Ed.). New York: Dover. Thus, thunder is the sign of an approaching storm, a bullet hole the sign of a shooting, a footprint the sign of a prey. Other signs are icons or abstract representations of another object—for example, the stylized image of a pedestrian on a traffic crossing light. Yet other signs are symbols that have a purely arbitary relationship to another object. Again, to use traffic signs as an example, think of how a red octagon means “stop” and a yellow inverted triangle means “yield.”

The most common symbols of all, of course, are words. Consider: the word “dog” has no inherent relation to the actual animal. Instead, as C. K. Ogden and I. A. Richards famously pointed out, the word “dog” may connote a friendly pet to one person and a dangerous beast to another. To explain how words work, they proposed a triangle of meaning.Ogden, C. K., & Richards, I. A. (1923). The meaning of meaning. London: Kegan, Paul, Trench, Trubner. They theorized that meaning emerges from the interplay between a referent (in our example, a dog), a symbol (the word “dog”), and the reference (what a person thinks when he or she hears the word). As such, the meaning of “dog,” whether a cute pet or dangerous animal, resides not in the word but, rather, in the mind of the person. In this way, as Robert Craig observed, semiotic theorists regard communication as a process of “intersubjective mediation by signs.”Craig, op. cit. In other words, the meaning of a thing is subjective for each person. Thus, as we communicate about that thing, there is an encounter between the different meanings we each carry. The encounter is mediated by a sign—whether the sign is a word or an image—and that sign makes it possible for some meaning, at least, to be shared between communicators.

The annual employee recognition luncheon is replete with signs and symbols. In addition to the many words that are used, shared meaning is created by the symbol of the award certificates and plaques, by the printed program with elegant cursive script, by the cake and the balloons with congratulatory messages, by the round tables that were set up rather than the room’s usual conference seating, by the festive centerpieces on the tables, by the company posters and slogans posted on the walls, by the formal business attire of the executives who presented the awards, and by the large company logo that is hung on the podium and printed on items ranging from table napkins to tee shirts. All of these symbols enable important meanings—about company values, about employee loyalty, about labor-management relations—to be communicated and shared by dozens of people, even though each brings his or her own subjective thoughts to the event.

Finally, the company itself becomes a symbol as it takes on a distinctive corporate image. Roland Barthes equated this kind of “second-order” symbol to mythmaking and gave the French national flag as an example.Barthes, R. (1972). Mythologies (A. Lavers, Trans.). New York: Hill and Wang. In the same way, Apple Corporation has come to symbolize high-tech innovation, a corporate image that instills its employees with a strong sense of organizational identity. By contrast, government agencies are often seen as bureaucratic and wasteful so that administrators must work hard to imbue their employees with a countervailing image of public service. The same semiotic process is at work as the college or university you attend strives to symbolize learning (if teaching is emphasized), or discovery (if research is emphasized), or opportunity (if career training is emphasized), or advancement (if nontraditional student are emphasized).

Critical Tradition

After witnessing or hearing about the employee awards luncheon, a criticalA scholarly tradition that theorizes communication as discursive reflection, or reflection on the ways that discourses create dominant and marginalized voices. theorist would likely ask who decided that employee loyalty would be the only value recognized and the only value which deserved a special annual celebration. The decision, of course, was made by the dominant interests who hold power in the organization. The luncheon reifies their interests (by establishing loyalty to the company as a taken-for-granted part of organizational life) and universalizes their interests (by equating management interests with “company interests” so that other interpretations seem irrational). Even though employees are expected to be loyal in order to gain approval, the company has no corresponding obligation of loyalty to the employees and may lay them off as needed. Not only is this proposition tacitly accepted—but to suggest that a second luncheon be held, to make a public accounting of the company’s loyalty to its workers, would seem irrational. So would the suggestion that workers, rather than the human resources director, should plan the annual luncheon and decide what values should be recognized and what awards given. Yes, the employees are permitted by the leadership to plan a humorous “awards” segment—but that is only a parody, a way to control workers by giving them a sense of participation without any real substance.

Then, too, a critical theorist would point out how the awards luncheon, by celebrating only those employees who have served long term, actually silences the voices of traditionally marginalized workers. Historically underrepresented groups—women, persons of color, persons with disabilities, the working poor—have often lacked the access to acquire skills which would make them promotable in the corporate world. Because they are disproportionately placed in low-wage jobs, they are the first to be laid off or shunted into temporary work. Yet they do work that the company needs. Why is there no event to celebrate their contributions? Instead, the emphasis on longevity only marginalizes them further.

This all happens because the system follows an ideology that, in ways made to seem natural and inevitable, structures power relations to favor some at the expense of others. Stanley Deetz has described “managerialism” as an ideology that systematically distorts communication to produce a “discursive closure” that renders alternative views difficult to express or even think.Deetz, 1992, op. cit. The task of critical scholars is to “denaturalize” unjust ideologies and structures that are taken for granted, exposing them to resistance and discussion, and thereby reopening choices and possibilities the system had foreclosed. Thus, critical scholarship infuses research with action.

Rhetorical

For the last of the seven traditions in communication theory, we come to the oldest. More than 2,300 years ago Aristotle wrote The Rhetoric and gave us, as many believe, the world’s first systematic treatment of human psychology.For example, Heidegger, M. (1962). Being and Time (J. Macquarrie & E. Robinson, Trans.). San Francisco: Harper and Row; pg. 178. He lived in Athens, one of the democratic city-states of ancient Greece where citizens publicly stated their cases in the assemblies and courts. Alarmed that some used oratory for personal gain rather than public good, Aristotle examined how speakers persuaded audiences and devised a theory and method of reasoned public address.Aristotle (2006). On rhetoric: A theory of civic discourse (2nd ed.) (G. A. Kennedy, Trans.). Oxford, UK: Oxford University Press. Today the phrase, “That’s just rhetoric,” connotes hollow or self-serving words. Aristotle had the same concern about public oratory. Thus rhetoricalA scholarly tradition that theorizes communication as the practical art of discourse and how persuasion is accomplished. theory, from classical times to the present, has concerned itself with the problem of how things get done. In other words, rhetorical theorists—including those who study organizational rhetoric—examine the processes by which speaker (or rhetor) and listeners move toward each other and find common grounds to go forward.

Studies of organizational rhetoric distinguish between external rhetoric aimed at stakeholders outside the organization and internal rhetoric aimed at employees. Mary Hoffman and Debra Ford classified four types of external rhetoric: to create and maintain an organization’s public identity, to manage issues, to manage risks, and to manage crises. Internal rhetoric, on the other hand, aims to align employees with organizational values and imperatives so they are motivated to do their jobs.Hoffman, M. F., & Ford, D. J. (2009). Organization rhetoric: Situations and strategies. Thousand Oaks, CA: Sage. Thus, the rhetoric of the annual employee recognition luncheon is internal, an attempt by management to find common ground with employees and persuade them to adopt company values. After the luncheon, the company will engage in external rhetoric as the corporate communications office issues a press release that, when carried by local media, will hopefully reinforce the company’s image as a great workplace that inspires employee loyalty.

Rhetorical theory offers many avenues for analyzing the speeches heard at the awards luncheon. The classical theory of Aristotle, for example, holds that speakers must invent a persuasive argument, effectively arrange its points, word it an appropriate style, and deliver it in a suitable manner, while drawing on a memory of phrases, stories, and ideas to extemporaneously flesh out the argument for a given occasion or audience. Today we call this method the five canons of rhetoric. Yet to be compelling, arguments must be grounded in the shared topoi or mental topology of rhetor and audience. Thus, if everyone agrees that profit is good for both management and labor, then speeches at the awards luncheon can extol honorees for their contributions to the bottom line. But if the organization is nonprofit—like the college or university you attend—then arguments based on profitmaking would fall flat. Aristotle also theorized that artful rhetors can employ three types of proofs: logic (logos), emotion (pathos), and speaker credibility (ethos). Executives who spoke at the luncheon likely tried all three by stating how loyal employees are rewarded (logic), how such employees’ dedication is admirable (emotion), and how management can be trusted and believed (speaker credibility).

In recent decades, scholarly interest in rhetorical theory has grown and proposals for a “new rhetoric” have gained wide acceptance. Kenneth Burke held that persuasion cannot occur without identification; the task of the rhetor is “consubstantiation,” or a sharing of substances, with the audience.Burke, K. (1950). A rhetoric of motives. New York: Prentice-Hall. (Thus, company leaders could persuade employees to be loyal only if the audience felt that executives could understand and sympathize with their concerns.) Chaim Perelman contended that persuasion cannot occur without presence; the rhetor must highlight “elements on which the speaker wishes to center attention in order that they may occupy the foreground of the hearer’s consciousness . . . against the undifferentiated mass of available elements of agreement.”Perelman, C., & Olbrechts-Tyteca, L. (1969). The new rhetoric: A treatise on argumentation. South Bend, IN: University of Notre Dame Press; pg. 142. (Thus, management hopes its appeal for loyalty is enhanced by staging a special yearly event.) Walter Fisher contrasted a rational-world paradigm of persuasion through logic with a narrative paradigm in which audiences are persuaded by stories that ring true with their lived experiences and the “good reasons” validated by their communities.Fisher, W. (1987). Human communication as narration: Toward a philosophy of reason, value, and action. Columbia: University of South Carolina Press. (Thus, the awards luncheon will foster loyalty only if executives can tell a story that resonates with the lives of employees.) And contemporary rhetorical scholars are recognizing the materiality of rhetoric as it “not only helps to produce judgments about specific issues, it also helps to produce or constitute a social world.”Jasinksi, J. (2001). Sourcebook on rhetoric: Key concepts in contemporary rhetorical studies. Thousand Oaks, CA: Sage, 2001; pg. 192. (Thus, the rhetoric of the awards luncheon aims not only to persuade but, leaders hope, produce an organizational culture whose logics favor employee loyalty.)

Key Takeaways

  • Three models for how communication functions have been proposed: linear, interactional, and transactional. The linear model holds that a message travels in a straight line from its source, through a channel, and to its receiver. The interactional model holds that communication travels in a circle as a sender transmits a message and then the receiver responds with feedback; thus both parties become sender/receivers. The transactional model holds that sending and receiving occur simultaneously.
  • Seven traditions in communication theory have been identified by Robert Craig. The cybernetic tradition theorizes communication as information processing. The phenomenological tradition theorizes communication as dialogue and the experience of otherness. The sociopsychological tradition theorizes communication as expression, interaction, and influence rooted in human psychological processes. The sociocultural tradition theorizes communication as the production and reproduction of a social order, such as an organizational culture. The semiotic tradition theorizes communication as intersubjective mediation by signs, or the ways that a sign (including a word) or symbol of a thing mediates the different thoughts that people have about the thing and thus permit meaning to be shared. The critical tradition theorizes communication as discursive reflection, or reflection on the ways that discourses create dominant and marginalized voices. The rhetorical tradition theorizes communication as the practical art of discourse and how persuasion is accomplished.

Exercises

  1. Your class in Organizational Communication is itself a type of organization. Think about the communication that takes place in your class, whether the class is face-to-face or online. Would you say that communication between the students and the instructor is best explained as a linear, interactional, or transactional process? Explain your answer.
  2. In the subsection above entitled “What Communication Is: Seven Traditions,” we imagined how the annual employee awards luncheon could be explained, in turn, by each of the seven traditions. Now on your own, think of another communication scenario that occurs in organizations (perhaps in your college or university, such freshman orientation or the annual commencement ceremony) and then explain your scenario by each of the seven traditions.

4.3 Representative Modern Theories

Learning Objectives

  1. Understand the basic precepts of systems theory and Karl Weick’s theory of organizing and sensemaking.
  2. Understand the basic precepts of Giddens’s structuration theory and its applications made by Poole and McPhee to organizations.
  3. Understand how, according to feminist theory, organizations are gendered and a primary site for configuration of gender roles.

To this point, we have explored approaches to theorizing organizational communication rather than specific theories. Yet we believe that focusing first on approaches is important. To speak of “interpretive organizational theory,” or “critical organization theory,” or “postmodern organization theory” is not to speak of any one single theory. Rather, each is—along with the postpositive perspective—a general approach to the looking at the problem of organizational communication. Each approach is informed by its own ontology (belief about the how things exist), epistemology (belief about how things can be known), and axiology (belief about what is worth knowing). Then, out of their respective philosophical commitments of each approach emerge specific theories. In the remainder of this section we will describe important modern theories of organizational communication that have emerged from the different approaches. And by first grasping the underlying approaches and how each looks at the problem in a different way, we believe you will be better equipped to understand specific theories and “where they’re coming from.”

Before proceeding, though, we offer one last thought to help put matters into perspective. You have likely heard the popular catchphrase “paradigm shift.” It was coined half a century ago by Thomas Kuhn.Kuhn, T. (1962). The structure of scientific revolutions. Chicago: University of Chicago Press. A historian and philosopher of science, he noted that theories which scientists generally agreed were settled could suddenly be overturned. These “scientific revolutions” were not always due to a new discovery, but to a new way of looking at the problem. The old paradigm closed off alternative approaches but, over time, some scientists became dissatisfied until momentum built for a paradigm shift. Kuhn’s thesis has also been applied to the social sciences—and the domain of organizational studies provides an excellent case in point. What is now called the postpositive approach dominated the field into the 1970s, until some organizational communication researchers became interested in the concept of organizational culture and felt constrained by the postpositive paradigm. New ways of looking at the problem were needed.

A generation or two later, organizational communication research has spawned four paradigms that are widely recognized by scholars. None can lay claim to being “the” dominant paradigm. Neither is the postpositive approach obsolete; if anything, it informs more research projects and more researchers than the other approaches, and retains wide influence. This is especially so since postpositive research aims at prediction, which is valued by the corporate world as a key to improved management practices. Perhaps it is more accurate to say that organizational communication research has split into four communities, each with its own paradigm. If so, we believe this is unfortunate. The specific theories described in this section illustrate the innovative work being done, on important problems of contemporary organizational life, through different approaches to organizational communication—and through blending aspects of those approaches. This should persuade us that each approach has something to contribute. Having multiple paradigms in play, as we do today, presents the field with a unique opportunity.

Postpositive Approach: Systems Theory

Systems theoryA theory based on the metaphor of the organization as a biological organism, so that the organization is seen as an open system that interacts with its environment in order to acquire the resources it needs to survive and grow. offers a good illustration of how organizational communication research from a postpositive perspective has continued to develop and even incorporate insights from other approaches. The story of systems theory begins in the mid 1950s when, as we saw in Chapter 3 "Classical Theories of Organizational Communication", the heyday of classical management theory had passed and the human resources approach was ascendant. In 1956 the Canadian biologist Ludwig von Bertalanffy first published his “general system theory” which proposed that traits found in biological systems could be applied to any system.Von Bertalanffy, L. (1968). General system theory: Foundations, development, applications. New York: Braziller. A decade later, the notion of applying the theory to organizations was popularized in an influential book by Daniel Katz and Robert Kahn.Katz, D., & Kahn, R. L. (1966). The social psychology of organizations. New York: Wiley. The old metaphor of the organization as a machine was replaced by the metaphor of a biological organism. As a result, the conception of the organization as a closed system was replaced by that of an open systemA system that is open to its surrounding environment, as opposed to a closed system that is not. A closed system is only concerned with input and output, whereas an open system encompasses input, throughput, and output.. Where a machine operates on its own, a biological organism can only survive by interacting with and gathering inputs from its surrounding environment. Thus, compared to the input-output of a machine, the operations of a biological organism involve input-throughput-output (a concept we encountered in Chapter 1 "Introduction to Organizational Communication"; see Figure 1.1).

Through systems theory, other princples from biology have been applied to organizations. Like an organism, an organization is not an undifferentiated hodgepodge of parts but a system with a hierarchical orderingThe notion is system theory which states that an organization is not a mass of undifferentiated parts, but that the parts are ordered in some way.. Further, these ordered parts are interdependentThe notion in systems theory that the parts of the system depend on one another in order to properly function. since they rely on one another to properly function. Being interdependent, the system enjoys the property of holismThe notion in systems theory that the whole is greater than the sum of its parts. or of being greater than the sum of its parts. But since the parts of the system must work together, feedbackThe name given in systems theory to internal communication that corrects deviation and spread information to help parts function better. is required both to correct deviations and spread information that fosters growth. In addition to communication within the system, the organism requires exchangeThe name given in systems theory to external communication that, in exchange for outputs, acquires the inputs or resources the organization needs to grow. with its environment. Unless the system exchanges outputs in order to acquire the inputs it needs to function, then the organization will feed on itself and eventually die. But because the system is open and its boundaries are permeableThe notion in systems theory which states that the boundaries of an organization are permeable so that exchanges can occur with the surrounding environment., the organization benefits from negative entropyIf an organization acquires no resources from the surrounding environment then it will feed on and eventually exhaust itself; this is called entropy. But a healthy organization that interacts with its environment can acquire resources and thus grow or experience negative entropy.—that is, because needed resources can pass freely into the system, it can grow. Yet to handle inputs from the environment, the system needs the requisite varietyThe notion in systems theory which states that an organization must a level of variety that is sufficient to deal with the level of complexity in its environment. to do so. In other words, the system’s complexity must be a match for the complexity of its inputs. If it does then the system will possess the trait of equifinalityThe ability of an organization to achieve a given goal in more than one way., which means the organization has multiple means to achieve a given goal and need not depend on only one option.

Perhaps the most influential single theory to emerge from the systems approach was proposed in the 1960s by Karl Weick and refined by him in the next three decades.Weick, K. (1969). The social psychology of organizing. Reading, MA: Addison-Wesley; Weick, K. (1995). Sensemaking in organizations. Newbury Park, CA: Sage. The theory begins with the observation that an organization’s environment includes information as well as material resources. Since the late twentieth century, the information environment has grown increasingly complex. Many communication situations cannot be handled by routines and rules. Moreover, the organization and its members both shape, and are shaped by, the information environment in which they operate—a principle Weick borrowed from the interpretive approach. Because all these factors introduce what he called “equivocality,” the goal of organizational communication is equivocality reductionBecause modern organizations are confronted by an increasingly complex information environment then, according to Weick’s theory, they seek to reduce the amount of equivocality (uncertainty) they experience.. To achieve the requisite variety needed to meet the challenges of a complex information environment, organization members’ natural response is the enactment of their own internal informational culture—again, a notion taken from the interpretive approach, and yet also an aspect of the interdependency predicted by systems theory. Next, Weick proposed a concept drawn from Darwin’s theory of evolution and natural selection. Though organization members have enacted an information environment, they each bring different interpretations of what that environment means. Thus, a part of the organizing process is selection of the best interpretations and then their retention to guide future enactments and selections. This collective process of enactment, selection, and retentionIn Weick’s theory, organizations respond to equivocality in their environments by enacting their own information system, selecting their best responses for reducing equivocality, and retaining them to guide future responses. is, in Weick’s model, called retrospective sensemakingName given in Weick’s theory to the process of enactment, selection, and retention by which organization members make sense of their environment.. Organization members muddle through complexities, perceive over time what works, and collectively reduce equivocality and make sense of their workplace. Thus, Weick has constructed a theory of organizing that is rooted in systems theory and follows a pospositive research agenda of observing and measuring aggregate behaviors. And yet his theory usefully draws on interpretive principles about the social constructedness of collective environments.

Interpretive Approach: Structuration Theory

Weick’s theory of organizing blends interpetive perspectives into a systems theory. But a key systems concept—that the parts of a system are interdependent—is given a new, interpretive twist through structuration theoryA theory proposed by Giddens to answer the question: Do people have free will or are they determined by their environments? He theorized that structure and agency are not a dualism but a duality. That is, people’s actions produce structure but, by acting within a structure, they also perpetuate or reproduce it. and its applications to organizational life. In traditional systems theories about organizations, the parts of the system are the various departments which have been hierarchically ordered to comprise an organization. But in structuration theory, the systemIn structuration theory, a system is comprised not of parts (such as an organizaation’s various departments) but of human practices. is a system of human practicesIn structuration theory, patterns of activity which have meaning for participants., where practices are understood as patterns of activity which have meaning for participants. Thus, the organizational system is not the operations department, the marketing department, the accounting department, and so on. Rather, the organizational system is comprised of patterns of practices—from the way that sick leave is handled, to the way that purchase orders are processed, to the way that meetings are conducted. In structuration theory, then, “structure” is not used in the conventional sense of a hierarchy. Rather, structureIn structuration theory, the interrelationships between human practices. refers to the interrelationships between practices.

By basing notions of system and structure on practices, and defining practices as meaningful patterns, we can see how structuration theory reflects the interpretive focus on the social constructedness of human groupings. This move sprang from, as Marshall Scott Poole and Robert McPhee have related, the concerns of scholars in the 1960s who felt that the sociopsychological emphasis in communication studies did not adequately allow for communal effects. “[T]he properties of systems were most often cast as constraints on behavior that acted from outside the individuals involved,” such that individuals were not seen as agents involved in constructing those systems.Poole, M. S., & McPhee, R. D. (2005). Structuration theory. In S. May & D. K. Mumby (Eds.), Engaging organizational communication theory and research: Multiple perspectives (pp. 171–195). Thousand Oaks, CA: Sage; pg. 171. Then in the late 1970s and 80s, communication scholars discovered the work of British theorist Anthony Giddens.Giddens, A. (1979). Central problems in social theory: Action, structure, and contradiction in social analysis. Berkeley: University of California Press; Giddens, A. (1984). The constitution of society: Outline of the theory of structuration. Cambridge, MA: Polity. His theory of structuration resolved the structure-versus-agency debate with an innovative move. Giddens proposed that structure and agency are not “either/or” but are “both/and,” or as he put it, not a dualism but a duality. In other words, people create a structure through their actions—but they also perpetuate, or reproduceIn struction theory, this refers to way that people within a system perpetuate its structure by acting within that structure., the structure by acting within it. As Giddens explained, structure is both “a medium and an outcome” of social action. Structure not only constrains action but also enables it, even as action produces and reproduces the structure—and thus, we have the process of structuration which gives the theory its name. Individuals act within the structure’s enablements and constraints by drawing on shared rulesStructuration theory holds that individuals act within a structure by drawing on shared rules to guide their actions and by employing resources (whether material or nonmaterial) to take action. to guide their actions, as well as resources (whether material or nonmaterial) they can employ to take action. And as the process of structuration goes forward, a system of practices evolves which guides signification (how things are interpreted), legitimation (what is deemed moral and should be done), and domination (how power is distributed to get those things done). Nevertheless, people are mostly unaware that their actions are grounded in, and impact upon, larger structurational process because their actions and consequences are separated in space and time.

Poole and McPhee outlined how structuration theory might be generally applied to communication studies and then, in the early 1980s, began exploring its applications to organizational communication.McPhee & Poole (1980). A theory of structuration: The perspective of Anthony Giddens and its relevance for contemporary communication research. Paper presented at the Speech Communication Association annual convention, New York. First, they suggested that structuration can explain the formation of an organization’s climate (a concept discussed in Chapter 6 "Organizational Communication Climate, Culture, and Globalization") or its “collective attitude, continually produced and reproduced by members’ interactions.”Poole & McPhee (1983). A structurational analysis of organizational climate. In L. L. Putnam & M. Pacanowski (Eds.), Communication and organizations: An interpretive approach (pp. 195–220). Beverly Hills, CA: Sage; pg. 213. Climate emerges from a concept pool of shared terms and phrases that members use to describe the organization, culminating in a kernel climate as members adopt a commonly shared abstraction to capture their basic understandings of the organization, and then progressing into particular climates that guide members’ attitudes and actions in specific situations.McPhee, R. D. (1985). Formal structure and organizational communication. In R. D. McPhee & P. Tompkins (Eds.), Organizational communication: Traditional themes and new directions (pp. 149–177). Beverly Hills, CA: Sage.

Further developments in organizational structuration theory have asserted that organizational communication occurs at three centers of structuration: conception, implementation, and reception.McPhee, R. D. (1989). Organizational communication: A structurational exemplar. In B. Dervin, L. Grossberg, B. J. O’Keefe & E. Wartella (Eds.), Rethinking communication: Paradigm exemplars, Vol. 2 (pp. 199–212). Beverly Hills, CA: Sage; McPhee, R. D., & Zaug, P. (2000). The communicative constitution of organizations: A framework for explanation. Electronic Journal of Communication, 10, 1–16. Though much overlap and conflict can occur, top management typically dominates conceptual communication, middle management oversees implementation, and employees receive and enact what has been decided. These communications may be classed into four flows that are respectively concerned with membership negotiation (who can be a member), activity coordination (what members do), self-structuring (how activities are organized), and institutional positioning (how the organization differentiates itself from others). Meanwhile, structurational processes also operate at the individual level to drive organizational identity (a phenomenon addressed in Chapter 8 "Organizational Identity and Diversity"). Through an identity-identification duality, the more that organization members are linked with other members who share the same premises, the more they will all cultivate a like identity for themselves and, in turn, be self-actualized by relationships with likeminded individuals.Cheney, G., & Tompkins, P. K. (1987). Coming to terms with organizational identification and commitment. Central States Speech Journal, 38, 1–15; Scott, C. R., Corman, S. R., & Cheney, G. (1998). Development of a structurational model of identification in the organization. Communication Theory, 8, 298–335.

Giddens’s original stucturation theory addressed processes at the societal level and the institutions that societies create and sustain. A societal institution, he observed, can accrue and channel great power as it becomes a nexus for concentrating, organizing, controlling, and then projecting resources. As mentioned above, structurational processes govern how things are interpreted, what is deemed moral and should be done, and how power is distributed to do those things. This same attention to the ways in which power operates through a system has continued to be a concern for scholars who apply structuration theory to organizations. In looking back on two decades of theory development, Poole and McPhee believed, “Structuration theory . . . has the potential to bring a critical edge to the analysis of organizational systems because it charges scholars to look for the role of power and domination in structuring processes that underlie organizations.” Further, the theory “shows how organizations are created and sustained by human action and how, potentially, they can be changed.”Poole, M.S., & McPhee, R. D., 2005, op. cit., pg. 180. Thus, they concluded, “We hope future researchers will take a more critical stance in developing the future of the structurational perspective.”Ibid, p. 192.

Critical/Postmodern Approaches: Feminist Theory

Feminist theoryNot a single theory, but an approach to organizational communication scholarship that sees organizations as gendered and as sites for configuring gender roles. provides an apt subject to conclude this chapter on modern theories of organizational communication—and to conclude this section, which has highlighted ways that modern theories often blend perspectives from different approaches. First, a cohesive body of work in feminist organizational communication studies is a newer development, having emerged only since the 1990s. Second, despite its recent pedigree, feminist theorizing about organizational communication draws on diverse schools of thought which have enriched one another in innovative ways. Katherine Miller identified five distinct approaches to feminist theorizing;Miller, K. (2012). Organizational communication: Approaches and processes (6th ed.). Boston: Wadsworth. Marta Calas and Linda Smircich identified seven;Calas, M. B., & Smircich, L. (1996). From “the women’s point of view”: Feminist approaches to organization studies. In S. R. Clegg, C. Hardy & W. R. Nord (Eds.), Handbook of organization studies (pp. 218–257). Thousand Oaks, CA: Sage. and Michael Papa, Tom Daniels, and Barry Spiker recently identified eight.Papa, M. J., Daniel, T. D., & Spiker, B. K. (2008). Organizational communication: Perspectives and trends. Thousand Oaks, CA: Sage. Feminist theorizing that specifically addresses organizational communication is generally identified with the critical approach to organizations, although Dennis Mumby also identified a postmodern strain in feminist inquiry about organizational life.Mumby, D. K. (2001). Power and politics. In F. M. Jablin & L. L. Putnam (Eds.), The new handbook of organizational communication: Advances in theory, research, and methods (pp. 585–623). Thousand Oaks, CA: Sage.

While feminist theorizing has generated a diverse body of research, some common themes emerge. Yet before describing these, let us review some ideas that retain a large influence on popular culture (so that you have probably heard about them) but do not represent current directions in feminist organizational communication studies. Karen Lee Ashcraft has recounted how research in the 1980s and 90s frequently focused on sex differences.Ashcraft, K. L. (2005). Feminist organizational communication studies: Engaging gender in public and private. In S. May & D. K. Mumby (Eds.), Engaging organizational communication theory and research: Multiple perspectives (pp. 141–169). Thousand Oaks, CA: Sage. Women were depicted as “different” than men. Even if a “feminine style” of communication and leadership was celebrated as valuable, the notion nevertheless perpetuated a stereotype. Moreover, “feminine style” and “women’s concerns” were largely equated with those of middle-class white women. Finally, the workplace was seen as neutral territory into which individuals brought their sex differences and societal prejudices; organizations were simply places where people played out gender issues imported from the outside. Two decades of feminist organizational communication scholarship has altered this picture. The workplace is no longer viewed as neutral; instead, organizations are seen as profoundly gendered institutions and a primary source of gendering (i.e., configuring gender roles) in contemporary society.

To explain this phenomenon, let us begin with the basic concept of “male” and “female.” The concept inherently lends itself to “either/or” thinking or what scholars call a binaryAn opposition such as male/female, cause/effect, rational/emotional, leader/follower, win/lose, public/private. Feminist theory holds that binary thinking in organizations leads to the domination of “masculine” values such as competition over “feminine” values such as cooperation. mode of thought. Another binary common to Western society is the “mind/body” dualism which holds that the mind is nonmaterial and the body is physical. A tendency to think in terms of binaries, feminist scholars have shown, suffuses modern organization in ways that (as critical theory tells us) work simultaneously to make “masculine” values dominant and yet hide that domination by making it seem natural. For example, the modern bureaucratic organizations is based on hierarchical ordering—which privileges the abstract (a “masculine” value) over the personal (a “feminine” value), establishes the workplace on the basis of individual categorization rather than egalitarian cooperation, and fosters a management/labor binary.Gilligan, C. (1982). In a different voice: Psyhological theory and women’s development. Cambridhe, MA: Harvard University Press. Hierarchy promotes linear modes of thinking and communication so that organizations extol a cause/effect binary as the only rational way to make decisions, rather than a holistic approach.Buzzanell, P. M. (1994). Gaining a voice: Feminist organizational communication theorizing. Management Communication Quarterly, 7, 339–383. Indeed, the rational/emotional binary is another mode of thought that gives a “masculine” gendering to organizations.Mumby, D. K., & Putnam, L. L. (1992). The politics of emotion: A critical reading. Academy of Management Review, 17, 465–486. Likewise, the leader/follower binary, which is inherent to hierarchy, is patriarchal and fosters a dependency that “feminizes” workers.Ferguson, K. (1984). The feminist case against bureaucracy. Philadelphia: Temple University Press. Conflict management is another area where “masculine” values reign; even when individuals agree to set aside the win/lose binary, settling their differences through negotiation privileges the skill of bargaining.Putnam, L. L., & Kolb, D. M. (2000). Rethinking negotiation: Feminist views of communication and exchange. In P. M. Buzzanell (Ed.), Rethinking organizational and managerial communication from feminist perspectives (pp. 76–104). Thousand Oaks, CA: Sage. Modern organizations even configure female bodies according to masculine values, as a woman’s “professional body” is expected to be trim and thus show discipline.Tretheway, A. (1999). Disciplined bodies: Women’s embodied identities at work. Organization Studies, 20, 423–450; Tretheway, A. (2000). Revisioning control: A feminist critique of disciplined bodies. In P. M. Buzzanell (Ed.), Rethinking organizational and managerial communication from feminist perspectives (pp. 107–127). Thousand Oaks, CA: Sage; Tretheway, A., Scott, C., & LeGreco, M. (2006). Constructing embodied organizational identities: Commodifying, securing, and servicing professional bodies. In B. Dow & J. T. Wood (Eds.), Handbook of gender and communication (pp. 123–141). Thousand Oaks, CA: Sage. Finally, Weber’s ideal of the impersonal-but-fair bureaucracy ideal (see Chapter 3 "Classical Theories of Organizational Communication") is alive today in the public/private binary that governs most organizations. That “masculine” values are the standard is ensured by the rule that public workplace performance takes precedence of private concerns.Ashcraft, 2005, op. cit.

The connections between feminist theory and critical theory are clear. Today, a growing body of feminist research employs fieldwork and ideology critique to show how organizations reify and universalize “masculine” values. Recently, other feminist scholars have demonstrated how a postmodern approach to organizations can inform research by deconstructing how discourses of domination—such as male domination—have formed over time. An example is Ashcraft and Mumby’s study of airline pilots, a profession in which white males predominate. They documented how early aviators were romanticized as daredevils but, when the prospect of commercial aviation arose, celebrated “lady” pilots made aviation seem less intimidating. Then as commercial flights became a reality and the public worried about safety and reliability, the industry promoted the image of the fatherly (white) professional pilot in a crisp naval-style uniform.Ashcraft, K. L., & Mumby, D. K. (2004). Reworking gender: A feminist communicology of organization. Thousand Oaks, CA: Sage. Their project also explores issue of race and, as such, illustrates how in recent years the feminist approach to organizational communication has begun to encompass issues of race, class, sexuality, and ability.

As our look at systems theory, structuration theory and feminist theory affirms, modern theories of organizational communication are diverse. In addition, they afford opportunities for innovative, as well as blended, approaches to explaining problems of organizational life in the twenty-first century.

Key Takeaways

  • Systems theory is based on the metaphor of the organization as a biological organization. Its parts, though hierarchically ordered, are interdependent and the whole is greater than the sum of the parts. Feedback between parts spreads needed information. But an organization is as an open system with permeable boundaries, engaging in exchange with its environment to gather resources required for growth. Because its environment is complex, the organization must have the requisite variety to deal with that complexity. This is aided through equifinality or the ability to take multiple paths for achieving the same goal. Karl Weick’s theory of organizing holds that information is part of an organization’s environment. Since the information is increasingly complex, a goal of organizational communication is equivocality reduction through processes of sensemaking.
  • Structuration theory provides a new answer to the question: Do people have free will or are they determined by their environments? Anthony Giddens theorized that structure and agency are not a dualism but a duality. That is, people’s actions produce structure but, by acting within a structure, they also perpetuate or reproduce it. This is called structuration. Thus, structure is both an outcome of, and a medium for, social action. For Giddens, a system is comprised human practices that have meaningful patterns for participants. Poole and McPhee, and subsequent scholars, have applied the theory of structuration to explain the processes of organizational climate, organizational communication, and organizational identity.
  • Feminist scholarship on organizational communication encompasses a diversity of approaches and theories. However, they share a conception of the modern organization as being gendered (rather than a neutral site where sex differences and societal prejudices play out) and as a primary site in the modern world for configuring gender roles. The male/female distinction reflects a binary mode of thought that suffuses organizations. Other binaries found in organizations include cause/effect, rational/emotional, leader/ follower, win/lose, and public/private. Bureaucratic hierarchy “feminizes” workers by making them dependent, while basing organizational life on individual categorization rather than egalitarian cooperation.

Exercises

  1. Where classical theory is based on the metaphor of the organization as a machine, systems theory is based on the metaphor of the organization as a biological organism. Make a list of other possible metaphors that might be used and explain how each one can help us understand the ways that an organization works.
  2. In developing a theory of organizational structuration, Robert McPhee proposed that communication occurs in four flows: membership negotiation (who can be a member), activity coordination (what members do), self-structuring (how activities are organized), and institutional positioning (how the organization differentiates itself from others). Think of an organization to which you belong and then make a list of the types of communication that occur within each of the four flows. Can you identify any patterns or structures in these communication practices? Do these structures help people in the organization get their messages across and be understood? Or do they limit what people can say? Or both?
  3. Again, think of an organization to which you belong. Name some examples of binary thinking (e.g., cause/effect, rational/emotional, leader/follower, win/lose, public/private) you have observed. How does this binary thinking affect what you have experienced in the organization? Does this type of thinking tend to make “masculine” values (e.g., competition, individualism, being rational, showing no emotion, taking action) more favored than “feminine” values (e.g., cooperation, integration).

4.4 Chapter Exercises

Real World Case Study

The Walt Disney Company and its theme parks have drawn the interest of organization and management scholars for decades. Books and articles praising Disney management began appearing in the 1960s. The runaway 1982 bestseller, In Search of Excellence by Thomas Peters and Robert Waterman, remains in print and lauds the Disney organization as a “best example” of customer service and employee relations.Peters, T. J., & Waterman, R. H., Jr. (1982). In search of excellence: Lessons from America’s best companies. New York: Harper & Row. The high profile of the Disney theme parks in U.S. and global culture have prompted studies not only by industrial psychologists and management scientists, but by scholars who take interpretive, critical, and postmodern approaches to organizational communication.For example, see Boje, D. M. (1995). Stories of the story-telling organization: A postmodern analysis of Disney as “Tamara-land.” Academy of Management Review, 38, 997–1035; Van Maanen, J. (1991). The smile factory: Work at Disneyland. In P. J. Frost, L. F. Moore, M. R. Lewis, C. C. Lundberg & J. Martin (Eds.), Reframing organizational culture (pp. 58–76). Newbury Park, CA: Sage; Van Maanen, J. (1992). Displacing Disney: Some notes on the flow of organizational culture. Qualitative Sociology, 15, 5–25; Van Maanen, J., & Kunda, G. (1989). Real feelings: Emotional expression and organizational culture. Research in Organizational Behavior, 11, 43–103.

One innovative study, though conducted some 30 years ago, reads like today’s news. The Disneyland theme park in California was dealing with the economic effects of a recent recession. Since the park was founded in 1955, management had succeeded in building up an unusually close-knit organizational culture. So a 1984 strike by park employees, protesting a management proposal to freeze wages and reduce benefits, made national headlines. Two organizational communication researchers, Ruth Smith and Eric Eisenberg, decided to investigate these labor troubles by taking an interpretive approach to the Disney organizational culture.Smith, R. C., & Eisenberg, E. M. (1987). Conflict at Disneyland: A root-metaphor analysis. Communication Monographs, 54, 367-380. They interviewed managers from several departments, reviewed company documents, and found that management carefully cultivated the metaphor of Disneyland as a “drama” or “show.” Customers were “guests” and employees, as the “cast,” were expected to play their “roles” by talking in approved phrases that followed the “script.” Dress codes and grooming requirements were called “costuming.” The park’s “on-stage” and “back-stage” areas were clearly delineated.

Then Smith and Eisenberg interviewed striking workers and were surprised by what they discovered. The company founder, Walt Disney, had died in 1966. His successors worked diligently to carry on his legacy so that the show might go on. In fact, according to Smith and Eisenberg, management was so successful in cultivating this idea that the park employees also took satisfaction in being caretakers of the Disney legacy. And with Disneyland’s emphasis on family entertainment, park employees began to see their workplace as a “family.” When management was compelled by the recession to emphasize the bottom line, employees believed the company was forsaking the Disney legacy and violating the spirit of the “Disney family.” Management responded by suggesting that families go through hard times, but to no avail. The strike lasted 22 days, the union went public with its concerns, management implemented a separate wage scale for new employees, and the organizational culture was profoundly changed.

  1. Smith and Eisenberg took an interpretive approach for their research on the organizational culture of Disneyland by analyzing company documents and interviewing managers and employees. If you were a postpositive researcher, how might you have conducted surveys of Disneyland employees to supplement the interviews? Would such knowledge of aggregate responses, alone, have helped you understand the Disneyland culture? Or would you have needed to interpret the mindsets of individual managers and employees? In other words, could nomothetic research by itself have sufficed or was ideographic research essential?
  2. If you had worked with Smith and Eisenberg, how might you have extended their study through ethnographic fieldwork? What Disneyland management and employee activities and rituals might you have observed and participated in? Smith and Eisenberg analyzed the metaphors that managers and employees used in their interviews; managers emphasized a “drama” metaphor, while employees also added a “family” metaphor. How might ethnographic fieldwork—as you participated in and directly experienced Disneyland culture for yourself—have extended the findings?
  3. What might a critical organizational communication scholar have said about the culture that Smith and Eisenberg found at Disneyland? Many organization and management scholars have praised the Disney company. But a critical scholar might ask: Did management use its “show” discourse to reify and universalize its interests? Was this discourse a kind of technical reasoning to gain a desired managerial goal and make practical reasoning toward mutual consent seem irrational? Did the discourse distort employees’ consciousness to favor management? Did it distort communication so that all communicative action took place on managerial terms? And as a postmodern scholar might have asked, did the discourse “manufacture consent” so that workers willingly disciplined themselves? What do you think?
  4. Re-read the description in the case of the “show” discourse that governed the organizational culture at Disneyland. Then re-read the discussion in this chapter about the seven traditions in communication theory: cybernetic, phenomenological, sociopsychological, sociocultural, semiotic, critical, and rhetorical. Now try to explain the “show” discourse according to each tradition.
  5. Karl Weick’s system theory holds that people collectively “make sense” of their workplace by enacting responses to its complexities, selecting the best responses, and retaining those responses to guide future enactments and selections. Try to interpret the Disneyland “show” culture that Eisenberg and Smith discovered through this framework.
  6. Anthony Giddens’s structuration theory holds that even as people create a structure, they simultaneously perpetuate or reproduce the structure by acting within what the structure enables and what it constrains. Try to explain our case study through this framework. Robert McPhee’s application of structuration theory to organizations holds that structuration occurs differently at the executive, middle management, and employee levels. Does this help explain why Disneyland employees went on strike?
  7. What might a feminist organizational communication scholar have said about the culture that Smith and Eisenberg discovered at Disneyland? Does their description suggest that the organization was gendered? Do you see any evidence of binary thinking in the Disneyland culture that Smith and Eisenberg’s describe?

End-of-Chapter Assessment Head

  1. In Chapter 4 "Modern Theories of Organizational Communication" we learned that theorists must make decisions about ontology, epistemology, and axiology. Select the answer below that gives the definitions of these three terms in the order of ontology, epistemology, and axiology.

    1. how things are known; what is worth knowing; how things exist
    2. what is worth knowing; how things exist; how things are known
    3. how things exist; what is worth knowing; how things are known
    4. how things exist; how things are known; what is worth knowing
    5. what is worth knowing; how things are known; how things exist
  2. The belief that a social phenomenon (such as an organization) has a subjective existence, and that it naturally tends toward order, are characteristic of which approach to organizations?

    1. Postpositive
    2. Interpretive
    3. Critical
    4. Postmodern
    5. Feminist
  3. The belief that a social phenomenon (such as an organization) is known by applying prior theoretical knowledge to the phenomenon, and that it naturally tends toward conflict, is characteristic of which approach to organizations?

    1. Postpositive
    2. Interpretive
    3. Critical
    4. Postmodern
    5. Functionalist
  4. The belief that a social phenomenon (such as an organization) exists independent of human perception, and that its structures are created through human agency, is characteristic of which approach to organizations?

    1. Postpositive
    2. Interpretive
    3. Critical
    4. Postmodern
    5. Social constructionist
  5. Which model depicts communication as a process by which communicators send messages/feedback simultaneously to one another?

    1. Sociopsychological
    2. Socicultural
    3. Linear
    4. Interactional
    5. Transactional

Answer Key

  1. D
  2. B
  3. C
  4. A
  5. E